You are on page 1of 143

Asian Math

Algebra I B

High School Assistance Foundation


Asian Math is a series of self-teaching books designed for junior/senior
high school students. It is designed according to the U.S. common core
standards and follows the Asian Mathematics Framework. This series of
books collects the essence of Asian teaching experiences and also absorbs
many American education theories. It aims to make math learning easy and
helps students improve math abilities, master mathematical thinking and
build solid mathematical foundation.

About the Author


The writing team of High School Assistance Foundation.
High School Assistance Foundation is a Nonprofit, 501(c)(3)
organization dedicated to providing high school students
with the knowledge and tools to pursue secondary education.

Published by High School Assistance Foundation


P.O. Box 203, Church Street Station.
New York, NY 10008 - 0203
Website: www.asianmath.org
Email: contactus@asianmath.org

Copyright © 2016 by High School Assistance Foundation


All rights reserved. Printed in the United States of America

ISBN 9 7 8 -0- 9 9 7 0 8 8 2 -7-4 (eBook)


ISBN 9 7 8 -0-9 9 7 0 8 8 2 -8-1 (Paper book)
Preface

Math is important for all students. Many students feel math is difficult. Modern
education theory believes the reason why students can't learn math well is not
intelligence but not taking appropriate teaching. We believe all students can
learn and master math easily.

To help U.S. high school students effectively learn math, we create a series of
tutoring and practice books according to the U.S. common core standards. This
series of books, Asian Math, is a kind of self-teaching materials. It follows the
Asian Mathematics Framework, and collects the essence of Asian teaching
experiences such as well-guided small step discovery learning, variation
teaching and variation practice, teaching based on knowledge network and
connections, orderly learning, etc. It also absorbs many American education
theories such as discovery learning, vivid real life situation introduction and
application, spiral learning theory, etc.

This series of books tries to involve students in the process of the occurrence of
knowledge. The small step exploration learning materials require students
observing, comparing, thinking, reasoning, summarizing and confirming by
immediate feedback. These will help students build up mathematical thinking,
improve math ability and strengthen the confidence of learning math.

How to use the book? When reading this series of books, try to answer each
question by yourself and check with the given answers. The answers to the
questions are at the end of the book. Hope everyone masters math efficiently
and easily. After reading, you are welcome to send us any feedback to improve
the book.

The writing team


High School Assistance Foundation.
Contents

Chapter 1 Polynomials and Factoring 1

1.1 Polynomials 2
1.2 Add and Subtract Polynomials 3
1.3 Multiplying Polynomials 13
1.4 Multiplying Special Cases I 19
1.5 Multiplying Special Cases II 21
1.6 Multiplying Special Cases III 26
1.7 Factoring Polynomials 30
1.8 Factoring Special Cases 36
1.9 Factoring Perfect Square Trinomial 42
Chapter Practice 45
Answer to Chapter 1 252

Chapter 2 Quadratic Equations 49

2.1 Quadratic Equations 49


2.2 Solving Quadratic Equations by Taking Square Roots 50
2.3 Solving Quadratic Equations by Completing the Square 54
2.4 The Quadratic Formula 63
2.5 The Discriminant 69
2.6 Real Word Problems About Quadratic Equations 72
Chapter Practice 81
Answer to Chapter 2 267
Chapter 3 Factoring Quadratics and 90
Solving Quadratic Equations

3.1 Factoring 𝒙𝟐 + 𝒃𝒙 + 𝒄 93
3.2 Cross Multiplication 96
3.3 Factoring Special Trinomials 100
3.4 Solving Quadratic Equations by Factoring 103
3.5 Factoring Quadratic Expressions By Solutions I 108
3.6 Factoring Quadratic Expressions By Solutions II 110
Chapter Practice 113
Answer to Chapter 3 280

Chapter 4 Quadratic Functions 117

4.1 Quadratic Functions 119


4.2 The Function of 𝒚 = 𝒂𝒙𝟐 127
4.3 The Function of 𝒚 = 𝒂𝒙𝟐 + 𝒌 137
4.4 The Function of 𝒚 = 𝒂(𝒙 + 𝒉)𝟐 140
4.5 The Function of 𝒚 = 𝒂(𝒙 − 𝒉)𝟐 + 𝒌 143
4.6 The Minimum and Maximum Value 150
4.7 The Function of 𝒚 = 𝒂𝒙𝟐 + 𝒃𝒙 + 𝒄 152
4.8 Finding the vertex using the formula 155
4.9 Graphing Quadratic Functions 158
4.10 Quadratic Functions and Quadratic Equations 168
4.11 Solving Quadratic Functions by Graphing 172
4.12 The Axis of Symmetry 179
4.13 Intersection Points of Graphs 184
4.14 Real Word Problems Involving Quadratic Functions 193
4.15 Modeling Quadratic Functions 204
4.16 Linear, Quadratic, and Exponential Models 212
4.17 Absolute Value Functions 218
Chapter Practice 235
Answer to Chapter 4 312
Chapter 4 Quadratic Functions

𝒇(𝒙) = 𝒂𝒙𝟐 + 𝒃𝒙 + 𝒄

1
Contents

4.1 Quadratic Functions

4.2 The Function of 𝒚 = 𝒂𝒙𝟐

4.3 The Function of 𝒚 = 𝒂𝒙𝟐 + 𝒌

4.4 The Function of 𝒚 = 𝒂(𝒙 + 𝒉)𝟐

4.5 The Function of 𝒚 = 𝒂(𝒙 − 𝒉)𝟐 + 𝒌


4.6 The Minimum and Maximum Value

4.7 The Function of 𝒚 = 𝒂𝒙𝟐 + 𝒃𝒙 + 𝒄


4. 8 Finding the Vertex Using the Formula
4. 9 Graphing Quadratic Functions
4.10 Quadratic Functions and Quadratic Equations
4.11 Solve Quadratic Equations by Graphing
4.12 The Axis of Symmetry
4.13 Intersection Points of Graphs
4.14 Real Word Problems Involving Quadratic Functions
4.15 Modeling Quadratic Functions
4.16 Linear, Quadratic, and Exponential Models
4.17 Absolute Value Functions

2
Chapter 4 Quadratic Functions

Parabola
Parabola is a U-shaped curve. When you throw
a basketball ( kick a soccer ball, throw a stone,
fire a missile or push a drinking water fountain),
it arcs up into the air and comes down again…..
following the path of a parabola!

All types of projectile motions can


be described by quadratic functions.

𝒚 = 𝒂𝒙𝟐 + 𝒃𝒙 + 𝒄

That is, a parabola is


a graph of a quadratic
function. And it is a
mirror-symmetrical
curve.

3
Quadratic Functions

What is a function? A function refers to a relationship between two quantities that


the change of one quantity is determined by the change of the other quantity. For
example: The area of a square is determined by the length of the side of the square.
When the side of the square is enlarged, the area of the square is correspondingly
enlarged. The function of this relationship is described by 𝒚 = 𝒙𝟐 . Here, we call
the length (𝒙) of the side of the square an independent variable. And we call the
area (𝒚) of the square a dependent variable.

Summary: A function means the dependent variable Y is determined by the


independent variable(s) X. For the function 𝒚 = 𝒙𝟐 , Y is the function of X.

In our life, there are many different functions.


For example: When you throw a stone into the
water, you can see the ripples spread and form
many circles with different sizes. The circles
get bigger as the radiuses get larger. That
means the circumferences of the circles are
determined by the radiuses. We can use a
function to describe this relationship: 𝒚 = 𝟐𝝅𝒓

We can also find the other function from the water ripples. The areas of the circles
get bigger as the radiuses get larger. That means the areas of the circles are also
determined by the radiuses. This relationship is described by a different function:
𝒚 = 𝝅𝒓𝟐 . Here, 𝒓 represents radiuses, 𝒚 represents areas. Y is the function of 𝒓.

𝒚 = 𝟐𝝅𝒓 is a linear function, which we have learned in Grade 8. While 𝒚 = 𝝅𝒓𝟐


is a non-linear function. Since it has the highest degree of 2, we call it a quadratic
function. Linear functions can be described by the standard form of 𝒚 = 𝒎𝒙 + 𝒃.
Next we will learn quadratic functions 𝒚 = 𝒂𝒙𝟐 + 𝒃𝒙 + 𝒄, where 𝒂 ≠ 𝟎.

4
Let’s start with the simplest quadratic function 𝒚 = 𝒙𝟐 , which is called the
quadratic parent function. Let’s graph the parent function first.

Step 1: Choose five or more points to graph the function and set the origin as
the center point. Since 𝒙 is an independent variable, we need find the 𝒚-values
corresponding to the 𝒙-values. Please fill in the following table of values.
Since the 𝒙-axis is horizontal, we use the horizontal table to show the values.

𝒙 ••• −𝟑 −𝟐 −𝟏 𝟎 𝟏 𝟐 𝟑 •••

𝒚 = 𝒙𝟐 ••• •••

Step 2: Plot each pair of the values (𝒙, 𝒚) on the following coordinate plane.
Step 3: Use a smooth curve to connect each points we plotted on the coordinate
plane. Then we get the graph of 𝒚 = 𝒙𝟐 shown on the following right.

-4 -3 -2 -1 0 1 2 3 4

The graph of the quadratic function 𝒚 = 𝒙𝟐 is a U-shape curve called a parabola.


The U-shaped curve is a mirror image. When you fold the U-shaped curve, the two
sides match exactly. This property is called symmetry. The line that divides the
U-shape curve into two matching halves is called the axis of symmetry.
The graph of the function 𝒚 = 𝒙𝟐 is symmetric about 𝒚-axis.
Think: Why the U-shaped curve is symmetric? Why it is a curve, not a straight line?

5
Average Rate of Change

The average rate of change of a function


is not new to us. In quadratic functions,
the average rate of change is not constant.
For example: John throws a stone up
into the air. When the stone goes up,
the speed of the stone is slowed down
by the gravity. After the stone reaches
its peak, the stone goes down and its
speed becomes more and more increased
due to the gravity. So the stone passes the different distances in the same time
intervals. We can see this from the above image on the right. That is why the
average rate of change of a quadratic function is not constant.

The average rate of change of a function is the slope of a line or a curve on


a given range, which is the change in the y-value divided by the change
in the x-value for the two distinct points on the graph.

The average rate of change can be


calculated by the following formula.

The average rate of change =


𝑹𝒊𝒔𝒆 ∆𝒚 𝒚𝟐 − 𝒚𝟏
Slope = = =
𝑹𝒖𝒏 ∆𝒙 𝒙𝟐 − 𝒙𝟏

Delta Notation
∆= “Change in”

6
Example 1: The graph of the quadratic function 𝒚 = 𝒙𝟐 is shown below.
Find the average rates of change between the intervals.
(1) 1 ≤ x ≤ 2 (2) 2 ≤ x ≤ 3

Solution: (1) When 𝒙𝟏 = 𝟏, 𝒚𝟏 = 𝟏, which is the point of (1, 1).


When 𝒙𝟐 = 𝟏, 𝒚𝟐 = 𝟏, which is the point of (2, 4).
The average rate of change between the interval: 1 ≤ x ≤ 2
𝑹𝒊𝒔𝒆 ∆𝒚 𝒚𝟐 − 𝒚𝟏 𝟒−𝟏 𝟑
= = = = = =𝟑
𝑹𝒖𝒏 ∆𝒙 𝒙𝟐 − 𝒙𝟏 𝟐 −𝟏 𝟏

(2) For the two points of (2, 4) and


(3, 9) , 𝒙𝟏 =____, 𝒚𝟏 = ____;
𝒙𝟐 =____, 𝒚𝟐 = ____;
The average rate of change between
the interval 2 ≤ x ≤ 3
𝒚𝟐 − 𝒚𝟏 ____− ____
= = = _____
𝒙𝟐 − 𝒙𝟏 ____− ____

Think: Are the above two rates equal


to each other? _________

Example 2: The above is the graph of the quadratic function 𝒚 = 𝒙𝟐 .


(1) Find the average rate of change between the interval 1 ≤ x ≤ 3.

Solution: (1) 𝒙𝟏 = 𝟏, 𝒚𝟏 = 𝟏 , which is the point of (1, 1).


𝒙𝟐 = 𝟑, 𝒚𝟐 = 𝟗 , which is the point of (3, 9).
The average rate of change between the interval : 1 ≤ x ≤ 3
𝑹𝒊𝒔𝒆 ∆𝒚 𝒚𝟐 − 𝒚𝟏 𝟗−𝟏 𝟖
= = = = = =𝟒
𝑹𝒖𝒏 ∆𝒙 𝒙𝟐 − 𝒙𝟏 𝟑 −𝟏 𝟐

(2) Find the average rate of change between the interval 0 ≤ x ≤ 3.


𝒚𝟐 − 𝒚𝟏 ____− ____
Slope = = = _____
𝒙𝟐 − 𝒙𝟏 ____− ____

7
Question 1: Why the shape of quadratic function is a curve, but not a straight line?

We are already familiar with linear functions and the concept of average rate
of change. In linear functions, the average rate of change (slope) is constant.
While in quadratic functions, the average rate of change is not constant. That
can be seen from the above two graphs. The left image is a graph of linear
function of 𝒚 = 𝟐𝒙 + 𝟏 , which has a constant slope of 2. The right image is a
graph of the quadratic function 𝒚 = 𝒙𝟐 , in which each horizontal unit change
(run) makes different changes of vertical movements (rise). That is the reason
why the graph of a quadratic function is a curve, but not a straight line.

Question 2: Why the U-shaped curve of the function 𝒚 = 𝒙𝟐 is symmetric?

We know that 𝟐𝟐 = 𝟒 and (−𝟐)𝟐 = 𝟒. Here, – 𝟐 is the opposite of 𝟐.


The squares of the opposite numbers are the same. We can see from the table
of values of 𝒚 = 𝒙𝟐 that the squares of each pair of opposite numbers of 𝒙
makes the same value of 𝒚 . That is the reason why the graph of 𝒚 = 𝒙𝟐 is
symmetric. For the same reason, if we take square roots of 𝒙𝟐 , we will get
two opposite roots. For example: When 𝒙𝟐 = 𝟒 , we get √𝒙𝟐 = ±√𝟒 = ±𝟐 .

8
Try 1: Please graph the quadratic function 𝒚 = −𝒙𝟐 on the coordinate plane.

Step 1: Find five or more points to graph it. And fill in the following table.

𝒙 ••• −𝟑 −𝟐 −𝟏 𝟎 𝟏 𝟐 𝟑 •••

𝒚 = −𝒙𝟐 ••• •••

Step 2: Plot each pair of the values (𝒙, 𝒚) on the coordinate plane.
Step 3: Use a smooth curve to connect each points we plotted on the coordinate
plane. Then we get the graph of 𝒚 = −𝒙𝟐 shown below.

-4 -3 -2 -1 0 1 2 3 4

Try 2: What is the difference between the graphs of 𝒚 = 𝒙𝟐 and 𝒚 = −𝒙𝟐 ?


The graph of the function 𝒚 = −𝒙𝟐 is also a parabola, and it opens __________.
The graph of the function 𝒚 = 𝒙𝟐 is a parabola that opens __________.
(A) upward (B) downward

Try 3: What is the same between the graphs of the two above functions?
First, the shapes of the parabolas continues up and out, in both directions, forever.
Second, They are both symmetric about ______ . They all pass through ________.
The origin is the _______ point of the function 𝒚 = 𝒙𝟐 and is the _______ point of
the function 𝒚 = −𝒙𝟐 . The highest or lowest point of a parabola is the vertex
of the parabola, which is on the axis of symmetry.
(A) 𝒙-axis (B) 𝒚-axis (C) highest (D) lowest (E) the origin

9
Question 3: Why the graph of the function 𝒚 = −𝒙𝟐 opens downward?

We can see this from the following table of values. In the function 𝒚 = −𝒙𝟐 , all
the values of 𝒚 are negative numbers except 0, which shows that except the origin,
all the plotted points on the coordinate plane will under the x-axis. And they are
all opposite numbers of the 𝒚-values of the function 𝒚 = 𝒙𝟐 , that states the graph
of 𝒚 = −𝒙𝟐 is the opposite and reflection of the graph of 𝒚 = 𝒙𝟐 across the x-axis.
So it opens downward.

𝒙 ••• −𝟑 −𝟐 −𝟏 𝟎 𝟏 𝟐 𝟑 •••

𝒚 = 𝒙𝟐 ••• 𝟗 𝟒 𝟏 𝟎 𝟏 𝟒 𝟗 •••

𝒚 = −𝒙𝟐 ••• −𝟗 −𝟒 −𝟏 𝟎 −𝟏 −𝟒 −𝟗 •••

From the other point of view, since the


values of 𝒚 in the function 𝒚 = −𝒙𝟐
are all negative numbers except 0,
here 𝒚 represents negative numbers.
𝒚 = −𝒙𝟐 is actually saying −𝒚 = 𝒙𝟐 .
𝒚 = −𝒙𝟐 is the opposite of 𝒚 = 𝒙𝟐
and its graph opens downward.

The vertex of the parabola is a turning point of a quadratic function where the
function turns. In the above two functions, the turning points are the origin (0,0).
In the function 𝒚 = 𝒙𝟐 , its graph changes from being decreasing to increasing as
the 𝒙-values increasing, just like an airplane dives and then goes up. The turning
point is the lowest point. The function 𝒚 = −𝒙𝟐 is opposite. It changes from
being increasing to decreasing as the 𝒙-values increasing, just like crossing over
a mountain turning at the mountain top.

10
The Function of 𝒚 = 𝒂𝒙𝟐

When the parent function 𝒚 = 𝒙𝟐 is multiplied by a constant 𝒂, we will get a new


function 𝒚 = 𝒂𝒙𝟐 . In our life, there are many functions in the form of 𝒚 = 𝒂𝒙𝟐 .
For example: The formula for the surface area of a cube: 𝑨 = 𝟔𝒔𝟐
𝟏
The Free Fall Formula: 𝒉 = 𝒈𝒕𝟐
𝟐
𝟏
The kinetic Energy Formula: 𝒌 = 𝒎𝒗𝟐
𝟐

Graph the functions 𝒚 = 𝒙𝟐 , 𝒚 = 𝟒𝒙𝟐


𝟏
and 𝒚 = 𝒙𝟐 . And compare them.
𝟒

From the right image, we can see that


the coefficient 𝒂 of the 𝒙𝟐 -term in a
quadratic function affects the width
of a parabola. The smaller | 𝒂 | is, the
wider the parabola is.

Try 1:
Please graph the functions 𝒚 = −𝒙𝟐 ,
𝟏
𝒚 = −𝟑𝒙𝟐 and 𝒚 = − 𝒙𝟐 on the
𝟑
coordinate plane. And compare them.

The graph of _______ has the widest


Parabola. The graph of __________
has the narrowest parabola. Please
order them from narrowest to widest.

______________________________

11
Question 4: In the quadratic function 𝒚 = 𝒂𝒙𝟐 , why is it true that the smaller | 𝒂 |
is, the wider the parabola is?

Let’s compare the graphs of the two


𝟏
functions: 𝒚 = 𝒙𝟐 and 𝒚 = 𝒙𝟐 .
𝟒
𝟏
For the function 𝒚 = 𝒙𝟐 , the two
𝟒
horizontal units change (run) makes
1 unit change of vertical movements 4

(rise). While for the function 𝒚 = 𝒙𝟐 ,


1
the two horizontal units change (run)
makes 4 units change of vertical
movements (rise). That is the reason
why the smaller | 𝒂 | is, the wider the parabola is.

Try 2: Please compare the graphs of the two functions: 𝒚 = 𝒙𝟐 and 𝒚 = 𝟒𝒙𝟐 .
And show that the greater | 𝒂 | is, the narrower the parabola is.

Try 3: Below are the graphs of three functions 𝒚 = 𝒎𝒙𝟐 , 𝒚 = 𝒏𝒙𝟐 and 𝒚 = 𝒑𝒙𝟐 .
(1) Compare the absolute values of
the three coefficients, which one is
the largest? _______
Which one is the smallest? ______
(A) | 𝒎 | (B) | 𝒏 | (C) | 𝒑 |

(2) The three functions have the same


vertex and the same axis of symmetry.
The vertex is _______. The axis of
symmetry is the ________.
(A) (0, 0) (B) x-axis (C) y-axis

12
The graph of 𝒚 = 𝒂𝒙𝟐 is the graph of
𝒚 = 𝒙𝟐 stretched or compressed. The
leading coefficient 𝒂 can be seen as a
scale factor of 𝒙𝟐 .

(1) If 0 < | 𝒂 | < 1, the graph of 𝒚 = 𝒙𝟐


becomes wider or ‘fat’.
(2) If | 𝒂 | > 1, the graph of 𝒚 = 𝒙𝟐
becomes narrower or ‘skinny’.

Example 1:
𝟏 𝟏
If the scale factor 𝒂 is , then the function 𝒚 = 𝒙𝟐 is transformed to 𝒚 = 𝒙𝟐 .
𝟒 𝟒
𝟏 𝟐
The graph of 𝒚 = 𝒙𝟐 is compressed to the y-axis to get the graph of 𝒚 = 𝒙 .
𝟒
𝟏
The graph of 𝒚 = 𝒙𝟐 is wider than the graph of 𝒚 = 𝒙𝟐 .
𝟒

Example 2:
If the scale factor 𝒂 is 𝟒 , then the function 𝒚 = 𝒙𝟐 is transformed to 𝒚 = 𝟒𝒙𝟐 .
The graph of 𝒚 = 𝒙𝟐 is stretched to the x-axis to get the graph of 𝒚 = 𝟒𝒙𝟐 .
The graph of 𝒚 = 𝟒𝒙𝟐 is narrower than the graph of 𝒚 = 𝒙𝟐 .

Try 4: In the quadratic function 𝒚 = 𝒙𝟐 , if 𝒙𝟐 is multiplied by 𝟖 , then the


new function is _________. The graph of the new function is _______
than the graph of 𝒚 = 𝒙𝟐 .
(A) 𝒚 = 𝒙𝟐 (B) 𝒚 = 𝟖𝒙𝟐 (C) wider (D) narrower

𝟏
Try 5: In the quadratic function 𝒚 = 𝟔𝒙𝟐 , if 𝟔𝒙𝟐 is multiplied by , then the
𝟐
new function is _________. The graph of the new function is ________
than the graph of 𝒚 = 𝟔𝒙𝟐 .
𝟏
(A) 𝒚 = 𝒙𝟐 (B) 𝒚 = 𝟑𝒙𝟐 (C) wider (D) narrower
𝟐

13
Summary:
For the graphs of the functions 𝒚 = 𝒂𝒙𝟐 ,
the vertex is the origin (0, 0) , and
the axis of symmetry is the y-axis.
If 𝒂 > 𝟎 , the parabola opens upward.
If 𝒂 < 𝟎 , the parabola opens downward.
The greater | 𝒂 | is, the narrower the parabola is.
The smaller | 𝒂 | is, the wider the parabola is.

Domain & Range:


The domain of a function is the set of all values of 𝒙 that will give values for 𝒚.
The range of a function is the set of all values of 𝒚. For example:

In the function 𝒚 = 𝒂𝒙𝟐 , the domain is all real numbers, ( −∞ < 𝒙 < +∞ ).
When 𝒂 > 𝟎, the graph of 𝒚 = 𝒂𝒙𝟐 opens upward. The minimum 𝒚-value is 0.
So the range is 𝒚 ≥ 𝟎. When 𝒂 < 𝟎, the graph of 𝒚 = 𝒂𝒙𝟐 opens downward.
The maximum 𝒚-value is 0. So the range is 𝒚 ≤ 𝟎.

Example 3: When 𝒙 = 𝟏, the function 𝒚 = 𝒂𝒙𝟐 has the value 𝒚 = −𝟐.


(1) Find the value of 𝒂. (2) Find the value of 𝒚, when 𝒙 = −𝟑.

Solution: (1) Since 𝒙 = 𝟏, 𝒚 = −𝟐 ,


𝒚 = 𝒂𝒙𝟐 −𝟐 = 𝒂(𝟏)𝟐 𝒂 = −𝟐

(2) Since 𝒂 = −𝟐, the original function is 𝒚 = −𝟐𝒙𝟐 .


When 𝒙 = −𝟑, 𝒚 = −𝟐𝒙𝟐 = −𝟐(−𝟑)𝟐 = −𝟐 • 𝟗 = −𝟏𝟖.
Answer: When 𝒙 = −𝟑, the value of 𝒚 is −𝟏𝟖.

Try 6: The graph of 𝒚 = 𝒂𝒙𝟐 passes through the point of (−𝟐, 𝟏𝟔).
(1) Find the value of 𝒂. (2) Find the value of 𝒚, when 𝒙 = 𝟓.

14
Example 4: An arch bridge crosses a river. The span length of the bridge is
300 meters and the clearance below is 45 meters shown as follows. Can you
find the quadratic function to describe the bridge?

Hint: Find the coefficient 𝒂 of the function 𝒚 = 𝒂𝒙𝟐 first. Then find the function.

How to Think: The bridge looks like a parabola. We can set up a coordinate plane
for the bridge. Let the vertex of the bridge be the origin of the coordinate plane,
then the quadratic function is 𝒚 = 𝒂𝒙𝟐 , and the middle of the span is the origin.
So the bridge has a length of 𝟏𝟓𝟎 meters and the height of −𝟒𝟓 meters away
from the origin, the graph of the bridge goes through the point of (𝟏𝟓𝟎, −𝟒𝟓).
Substitute 𝒙 = 𝟏𝟓𝟎 and 𝒚 = −𝟒𝟓 into the function. Then we can find the
coefficient 𝒂.

Solution: (1) 𝒚 = 𝒂𝒙𝟐


−𝟒𝟓 = 𝒂 • 𝟏𝟓𝟎𝟐
𝟏
𝒂=−
𝟓𝟎𝟎

𝟏
So (2) 𝒚 = 𝒂𝒙𝟐 = − 𝒙𝟐
𝟓𝟎𝟎

𝟏
Answer: The quadratic function of the bridge is 𝒚 = − 𝒙𝟐 .
𝟓𝟎𝟎

15
Example 5: Suppose we need design the supporting columns of the bridge (in
example 4). We want to set up the columns at the distance of 50 meters and 100
meters away from the center of the bridge. What are the heights of the two
supporting columns?

Solution: From example 4, we know that the function of the bridge is


𝟏
𝒚=− 𝒙𝟐 , we can find the y-value given the x-value. 50 m and 100 m are
𝟓𝟎𝟎
the given x-values. The corresponding y-values are the heights of the columns.
𝟏 𝟏
(1) When 𝒙 = 𝟓𝟎 , 𝒚=− 𝒙𝟐 = − (𝟓𝟎)𝟐 = −𝟓 m
𝟓𝟎𝟎 𝟓𝟎𝟎
𝟏 𝟏
(2) When 𝒙 = 𝟏𝟎𝟎 , 𝒚 = − 𝒙𝟐 = − (𝟏𝟎𝟎)𝟐 = −𝟐𝟎 m
𝟓𝟎𝟎 𝟓𝟎𝟎

Answer: The supporting column at the distance of 50 meters is 5 meters.


The supporting column at the distance of 100 meters is 20 meters.

Example 6: Suppose the water level of the river is 45 meters below the bridge
in the example 4. The width of the river is 300 meters. When the water level
rises up 5 meters, how wide is the river goes through the bridge?

How to Think: When the water level rises up 5 meters, the height of the water
level of the river on the coordinate plane is −𝟒𝟎 meters, which is the y-value.
We can find its related x-value, which is the half of the width of the river.
𝟏 𝟏
Solution: When 𝒚 = −𝟒𝟎 , 𝒚=− 𝒙𝟐 −𝟒𝟎 = − 𝒙𝟐
𝟓𝟎𝟎 𝟓𝟎𝟎

Solve for 𝒙: 𝒙 = 𝟏𝟎𝟎√𝟐 ≈ 𝟏𝟒𝟏. 𝟒 m


The width: 𝟏𝟒𝟏. 𝟒 ∗ 𝟐 = 𝟐𝟖𝟐. 𝟖 m
Answer: The river goes through the bridge has a width of about 282.8 meters.
16
Try 7: An arch bridge crosses a river shown below. The span length of the bridge
is 60 meters and the clearance below is 6 meters. Can you use a quadratic function
to describe the bridge? If the water level rises up 2 meters, how wide is the river
goes through the bridge?

Try 8: A suspension bridge crosses a river shown below. The length of the deck
is 240 meters and the height of the tower above the deck is 24 meters. Can you use
a quadratic function to describe the bridge? What is the length of hangers at the
distance of 30 meters and 60 meters away from the center of the deck?

17
Practice 1

1. The graph of the quadratic function 𝒚 = 𝒂𝒙𝟐 is shown below on the right.
From the graph, we can tell that: (Choice questions.)
(1) The coefficient 𝒂 _____ 0. The parabola opens ________.
(A) > (B) < (C) upward (D) downward
(2) The axis of symmetry is the ________ . The vertex is __________.
(A) x-axis (B) y-axis (C) (0, 0) (D) (1, 1)
(3) When 𝒙 =______, the minimum 𝒚-value occurs and it is __________.
(A) 0 (B) 1 (C) (0, 0)

(4) The domain of the function is ________.


The range of the function is __________.
(A) 𝒙 = All real numbers
(B) 𝒚 = All real numbers
(C) 𝒚>𝟎 (D) 𝒚<𝟎
(E) 𝒚 ≥ 𝟎 (F) 𝒚≤𝟎

𝟏
2. The graph of the quadratic function y = − 𝒙𝟐 has the following
𝟓
characteristics: (Choice questions.)
(1) The coefficient 𝒂 is ________. The parabola opens ________.
(A) positive (B) negative (C) upward (D) downward
(2) The axis of symmetry is the ________ . The vertex is __________.
(A) x-axis (B) y-axis (C) (0, 0) (D) (1, 1)
(3) When 𝒙 =______, the vertex occurs. The 𝒚-value of the vertex is _________,
which is the _____________ value.
(A) 0 (B) 1 (C) minimum (C) maximum
(4) The domain of the function is _______. The range of the function is _______.
(A) 𝒙 = All real numbers (B) 𝒚 = All real numbers
(C) 𝒚>𝟎 (D) 𝒚<𝟎 (E) 𝒚 ≥ 𝟎 (F) 𝒚 ≤ 𝟎

18
3. When ________, the graph of the quadratic function 𝒚 = 𝒂𝒙𝟐 (𝒂 ≠ 𝟎) is wider.
(A) 𝒂 is greater (B) 𝒂 is smaller
(C) | 𝒂 | is greater (D) | 𝒂 | is smaller

4. The graphs of the quadratic functions 𝒚 = 𝒙𝟐 and 𝒚 = −𝒙𝟐 have some same
characteristics that are ___________.
(A) The axis of symmetry is y-axis. (B) The vertex is the origin.
(C) The parabolas all open upward. (D) The parabolas all open downward.

5. In the quadratic function of 𝒚 = 𝟔𝒙𝟐 , given there are two opposites 𝒙-values,
the corresponding 𝒚-values will be ________. The two plotted points on the
coordinate plane will __________.
(A) the same value (B) the opposites
(C) reflect each other across the x-axis
(D) reflect each other across the y-axis

6. When the quadratic functions 𝒚 = 𝟐𝒙𝟐 and 𝒚 = −𝟐𝒙𝟐 have the same 𝒙-value,
Then the corresponding 𝒚-values will be ___________. The graphs of the two
functions on the coordinate plane will _____________.
(A) the same value (B) the opposites.
(C) reflect each other across the x-axis
(D) reflect each other across the y-axis
𝟏
7. For the graphs of the functions 𝒚 = 𝒙𝟐 , 𝒚 = 𝟑𝒙𝟐 , 𝒚 = −𝒙𝟐 , which graph
𝟐
has the narrowest opening? ___________
𝟏
(A) 𝒚 = 𝒙𝟐 (B) 𝒚 = 𝟑𝒙𝟐 (C) 𝒚 = −𝒙𝟐 (D) None
𝟐

8. The graph of the quadratic function 𝒚 = 𝒂𝒙𝟐 is shown below.


𝟏
If 𝒂 is multiplied by − , the graph of
𝟑
the new function is _____________.
(A) wider and opens upward
(B) wider and opens downward
(C) narrower and opens upward
(D) narrower and opens downward

19
9. True or False.
(1) The smaller | 𝒂 | is, the narrower the parabola is. ________
(2) The greater | 𝒂 | is, the narrower the parabola is. ________
(3) If 𝒂 is multiplied by a proper fraction (−1 < the fraction < 1), then the
graph of 𝒚 = 𝒂𝒙𝟐 will be compressed to the x-axis. __________
10. Which of the following characteristics of the graph of the quadratic function
𝒚 = 𝒂𝒙𝟐 (𝒂 ≠ 𝟎) is related to the coefficient 𝒂 ? ___________
(A) the axis of symmetry (B) the vertex.
(C) the opening width only (D) the opening direction only
(E) the width and the direction of the opening

11. When 𝒙 = −𝟑, the function 𝒚 = 𝒂𝒙𝟐 has the value 𝒚 = −𝟑𝟔.
(1) Find the value of 𝒂. (2) Find the value of 𝒚, when 𝒙 = 𝟓.

12. The graph of 𝒚 = 𝒂𝒙𝟐 passes through the point of (−𝟏, −𝟑).
(1) Find the value of 𝒂. (2) Find the function.
(3) Can you tell the opening direction of the graph?

13. The parabola of a quadratic function has the vertex of (0, 0).
And it passes through the points of (−𝟏, 𝟐). (1) Find the value of 𝒂.
(2) Find the function. (3) Can you tell the opening direction of the graph?

20
The Function of 𝒚 = 𝒂𝒙𝟐 + 𝒌

1. Below are the graphs of the quadratic functions 𝒚 = 𝒙𝟐 + 𝟐, 𝒚 = 𝒙𝟐 − 𝟐 and


𝒚 = 𝒙𝟐 on the coordinate plane.

What is the same among the graphs?


_______ What is different? _______
(A) They all open upward.
(B) They have the same vertex.
(C) They have the same axis
of symmetry, 𝒚-axis.
(D) They have the same shape
and same width of opening.

From the above graphs, we can see that they have the same shape, the same
opening direction and opening width, and the same axis of symmetry, 𝒚-axis.
The only difference among them is the vertices. Summary: We can shift the
graph of 𝒚 = 𝒙𝟐 up two units along the 𝒚-axis to get the graph of
𝒚 = 𝒙𝟐 + 𝟐. And we can shift the graph of 𝒚 = 𝒙𝟐 down two units
along the 𝒚-axis to get the graph of 𝒚 = 𝒙𝟐 − 𝟐.

2. Please graph the functions 𝒚 = 𝟐𝒙𝟐 + 𝟏, 𝒚 = 𝟐𝒙𝟐 − 𝟏 and 𝒚 = 𝟐𝒙𝟐 on the


coordinate plane.

Step 1: Fill in the following table of values.

𝒙 ••• −𝟑 −𝟐 −𝟏 𝟎 𝟏 𝟐 𝟑 •••

𝒚 = 𝟐𝒙𝟐 + 𝟏 ••• •••

𝒚 = 𝟐𝒙𝟐 ••• 𝟏𝟖 𝟖 𝟐 𝟎 𝟐 𝟖 𝟏𝟖 •••

𝒚 = 𝟐𝒙𝟐 − 𝟏 ••• •••

21
Step 2: Plot and graph 𝒚 = 𝟐𝒙𝟐 + 𝟏, 𝒚 = 𝟐𝒙𝟐 − 𝟏 and 𝒚 = 𝟐𝒙𝟐 .

(1) When 𝒙 =____, the function 𝒚 = 𝟐𝒙𝟐


has the minimum value, which is ____.
The vertex of the function is ________.
(2) When 𝒙 =___, the function 𝒚 = 𝟐𝒙𝟐 + 𝟏
has the minimum value, which is ____.
The vertex of the function is ________.
(3) When 𝒙 =___, the function 𝒚 = 𝟐𝒙𝟐 − 𝟏
has the minimum value, which is ____.
The vertex of the function is ________.

(4) True or false? For the same x-value, each y-value of 𝒚 = 𝟐𝒙𝟐 + 𝟏 is one more
than that of 𝒚 = 𝟐𝒙𝟐 . Each point of the graph 𝒚 = 𝟐𝒙𝟐 + 𝟏 is shifted up one
unit from the graph of 𝒚 = 𝟐𝒙𝟐 . So the two graphs have the same shape. Since
they have the same coefficient 𝒂, they have the same width of opening. _____

(5) Explain how is the graph of 𝒚 = 𝟐𝒙𝟐 − 𝟏 related to the graph of 𝒚 = 𝟐𝒙𝟐 .

3. If we shift the graph of 𝒚 = 𝟑𝒙𝟐 up eight units along the 𝒚-axis, then we can
get the graph of ___________. (A) 𝒚 = 𝟑𝒙𝟐 − 𝟖 (B) 𝒚 = 𝟑𝒙𝟐 + 𝟖
𝟏
4. To get the graph of 𝒚 = 𝒙𝟐 − 𝟔, we need shift the graph of _____________.
𝟐
𝟐
(A) 𝒚 = 𝒙 up six units (B) 𝒚 = 𝒙𝟐 down six units
𝟏 𝟏
(C) 𝒚 = 𝒙𝟐 up six units (D) 𝒚 = 𝒙𝟐 down six units
𝟐 𝟐
5. Fill in the table.
opening direction axis of symmetry Vertex (x, y)
𝒚 = 𝒙𝟐 + 𝟕
𝒚 = 𝟑𝒙𝟐 − 𝟕
𝒚 = −𝒙𝟐 + 𝟒
𝟐
𝒚 = − 𝒙𝟐 − 𝟒
𝟑

22
Example 1: Which function is shown in the graph?
𝟏 𝟏
(A) 𝒚 = 𝒙𝟐 + 𝟓 (B) 𝒚 = − 𝒙𝟐 + 𝟓
𝟒 𝟒
𝟏 𝟏
(C) 𝒚 = 𝒙𝟐 − 𝟓 (D) 𝒚 = − 𝒙𝟐 − 𝟓
𝟑 𝟑

Solution: Step 1: Since the parabola opens downward,


the function has a negative leading coefficient, 𝒂 < 𝟎 .
So we can eliminate the choices A and C.
Step 2: The vertex of the parabola is shifted up 5 units from the origin means
the whole parabola is shifted up 5 units. So the constant term is +𝟓. Look at the
choices. Choices A and B have a constant term of +𝟓. Since the choice A is
eliminated. The answer is B.

Try 6: Which of the following is the parabola of y = −𝟑𝒙𝟐 + 𝟐 ? _________

A B C

Try 7: Multiple Choices. Which functions


could be shown by the graph on the right? ____
𝟏
(A) 𝒚 = 𝟑𝒙𝟐 + 𝟐 (B) 𝒚 = 𝒙𝟐 + 𝟐
𝟑
𝟏
(C) 𝒚 = 𝟑𝒙𝟐 − 𝟐 (D) 𝒚 = 𝒙𝟐 − 𝟐
𝟑

23
The Function of 𝒚 = 𝒂(𝒙 + 𝒉)𝟐

1. Below are the graphs of the quadratic functions 𝒚 = (𝒙 + 𝟐)𝟐 , 𝒚 = 𝒙𝟐 and


𝒚 = (𝒙 − 𝟐)𝟐 on the coordinate plane.

What is the same among the graphs?


_______ What is different? _______
(A) They all open upward.
(B) The vertex
(C) The axis of symmetry
(D) They have the same shape
and same opening direction.

From the above graphs, we can see that they have the same shape, the same
opening direction and the same opening width, but they have different vertices
and different axes of symmetry. Summary: We can shift the graph of 𝒚 = 𝒙𝟐
to the left two units along the x-axis to get the graph of 𝒚 = (𝒙 + 𝟐)𝟐. And
we can shift the graph of 𝒚 = 𝒙𝟐 to the right two units along the x-axis to
get the graph of 𝒚 = (𝒙 − 𝟐)𝟐 .

2. Please graph the functions 𝒚 = 𝟐(𝒙 + 𝟏)𝟐 , 𝒚 = 𝟐(𝒙 − 𝟏)𝟐 and 𝒚 = 𝟐𝒙𝟐 on
the coordinate plane. Explain how they are related to the parabola of 𝒚 = 𝟐𝒙𝟐 .

Step 1: Fill in the following table of values.


𝒙 ••• −𝟑 −𝟐 −𝟏 𝟎 𝟏 𝟐 𝟑 •••

𝒚 = 𝟐(𝒙 + 𝟏)𝟐 ••• •••

𝒚 = 𝟐𝒙𝟐 ••• 𝟏𝟖 𝟖 𝟐 𝟎 𝟐 𝟖 𝟏𝟖 •••

𝒚 = 𝟐(𝒙 − 𝟏)𝟐 ••• •••

(1) When 𝒙 = 𝟐, 𝒚 = 𝟐𝒙𝟐 = ___; 𝒚 = 𝟐(𝒙 + 𝟏)𝟐 = ___; 𝒚 = 𝟐(𝒙 − 𝟏)𝟐 = ___.

24
Step 2: Plot and graph 𝒚 = 𝟐(𝒙 + 𝟏)𝟐 , 𝒚 = 𝟐(𝒙 − 𝟏)𝟐 and 𝒚 = 𝟐𝒙𝟐 .

(1) When 𝒙 =____, the function 𝒚 = 𝟐𝒙𝟐


has the minimum value, which is ____.
The vertex of the function is ________.
(2) When 𝒙 =___, the function 𝒚 = 𝟐(𝒙 + 𝟏)𝟐
has the minimum value, which is ____.
The vertex of the function is ________.
(3) When 𝒙 =___, the function 𝒚 = 𝟐(𝒙 − 𝟏)𝟐
has the minimum value, which is ____.
The vertex of the function is ________.

3. If we shift the graph of 𝒚 = 𝟓𝒙𝟐 to the left three units along the x-axis, then
we get the graph of ________. (A) 𝒚 = 𝟓𝒙𝟐 + 𝟑 (B) 𝒚 = 𝟓𝒙𝟐 − 𝟑

(C) 𝒚 = 𝟓(𝒙 + 𝟑)𝟐 (D) 𝒚 = 𝟓(𝒙 − 𝟑)𝟐


𝟏
4. To get the graph of 𝒚 = (𝒙 + 𝟗)𝟐 , we need shift the graph of ___________.
𝟒
𝟐
(A) 𝒚 = 𝒙 to the right nine units (B) 𝒚 = 𝒙𝟐 to the left nine units
𝟏 𝟏
(C) 𝒚 = 𝒙𝟐 to the right nine units (D) 𝒚 = 𝒙𝟐 to the left nine units
𝟒 𝟒

5. To get the graph of 𝒚 = −𝟑(𝒙 − 𝟔)𝟐 , we need shift the graph of ___________.
(A) 𝒚 = 𝒙𝟐 to the right six units (B) 𝒚 = 𝒙𝟐 to the left six units
(C) 𝒚 = −𝟑𝒙𝟐 to the right six units (D) 𝒚 = −𝟑𝒙𝟐 to the left six units

6. Fill in the table.


opening direction axis of symmetry Vertex (x, y)
𝒚 = (𝒙 + 𝟐 )𝟐
𝒚 = 𝟑(𝒙 − 𝟕)𝟐
𝒚 = −𝟒(𝒙 + 𝟓)𝟐
𝟐
𝒚 = − (𝒙 − 𝟒 )𝟐
𝟑

25
Try 7: Which of the following is the parabola of y = −𝟑(𝒙 + 𝟐 )𝟐 ? _________

A B C

Try 8: Which function is shown in the graph on the right?


(A) 𝒚 = 𝟒(𝒙 + 𝟑)𝟐 (B) 𝒚 = −𝟒(𝒙 + 𝟑)𝟐

(C) 𝒚 = 𝟒(𝒙 − 𝟑)𝟐 (D) 𝒚 = −𝟒(𝒙 − 𝟑)𝟐

Try 9: Match each function with its graph.


𝟏 𝟏
(1) 𝒚 = 𝒙𝟐 − 𝟑 ___ (2) 𝒚 = − 𝒙𝟐 − 𝟒 ___ (3) 𝒚 = − (𝒙 − 𝟒)𝟐 ___
𝟐 𝟐

A B C

Summary:
A translation moves a
figure up, down, left or
right. We are learning
the translation now.

26
The Function of 𝒚 = 𝒂(𝒙 − 𝒉)𝟐 + 𝒌

If we shift the graph of 𝒚 = 𝟐𝒙𝟐 three units to the left, we can get the graph of
𝒚 = 𝟐(𝒙 + 𝟑)𝟐 . If we further shift the graph of 𝒚 = 𝟐(𝒙 + 𝟑)𝟐 four units down,
then what we can get? Please write down the new function. _______________

The answer is the function of 𝒚 = 𝟐(𝒙 + 𝟑)𝟐 − 𝟒. We can check it. Below
is the graph we get by finding the y-values and plotting the points. The two
graphs are the same. From the graph, we find the vertex is (−𝟑, −𝟒 ).

𝒙 𝒚 = 𝟐(𝒙 + 𝟑)𝟐 − 𝟒
−𝟏 𝟒

−𝟐 −𝟐
−𝟑 −𝟒
−𝟒 −𝟐
−𝟓 𝟒

The axis of symmetry is the vertical line that goes through the vertex of a quadratic
function. The axis of symmetry of the above function is the line of 𝒙 = −𝟑.

27
Rewrite the function 𝒚 = 𝟐(𝒙 + 𝟑)𝟐 − 𝟒 as 𝒚 = 𝟐(𝒙 − (−𝟑))𝟐 + (−𝟒) ,
which is in the form of 𝒚 = 𝒂(𝒙 − 𝒉)𝟐 + 𝒌, where 𝒉 = −𝟑, 𝒌 = −𝟒.
Compared with the vertex in the graph, (𝒉, 𝒌) is the vertex (−𝟑, −𝟒 ).
So we call the form of 𝒚 = 𝒂(𝒙 − 𝒉)𝟐 + 𝒌 the vertex form.

Summary: The function 𝒚 = 𝒂(𝒙 − 𝒉)𝟐 + 𝒌 is called the vertex form


of a quadratic function. In the vertex form, the vertex is (𝒉, 𝒌).
The axis of symmetry of the function is the line of 𝒙 = 𝒉.

The 𝒉 represents a horizontal shift (how far left, or right, the graph has shifted).
The 𝒌 represents a vertical shift (how far up, or down, the graph has shifted).
Notice that the 𝒉 value is subtracted in this form, and that the 𝒌 value is added.
Given the function 𝒚 = 𝟕(𝒙 − 𝟓)𝟐 + 𝟗, the value of 𝒉 = 𝟓, 𝒌 = 𝟗.
Given the function 𝒚 = 𝟐(𝒙 + 𝟑)𝟐 − 𝟒, the value of 𝒉 = −𝟑, 𝒌 = −𝟒.

Try: 1. Find the vertex and draw the axis of symmetry on the graphs.

(1) The vertex is _______ (2) The vertex is _______


The axis of symmetry is ______ The axis of symmetry is ______

2. Given the function 𝒚 = 𝟖(𝒙 − 𝟐)𝟐 − 𝟒, the value of 𝒉 =______, 𝒌 =______.


The vertex of the function is ________. The axis of symmetry is _______ .
28
3. Given a function 𝒚 = 𝟐(𝒙 + 𝟑)𝟐 + 𝟔, the value of 𝒉 =______, 𝒌 =______.
The vertex of the function is ________. The axis of symmetry is _______ .

𝟐
4. Given a function 𝒚 = − (𝒙 − 𝟕)𝟐 + 𝟗, the vertex of the function is ______.
𝟓
The axis of symmetry of the function is ________ .
(A) (𝟕, 𝟗) (B) (−𝟕, 𝟗) (E) 𝒙 = 𝟕 (G) 𝒙 = 𝟗
(C) (𝟕, −𝟗) (D) (−𝟕, −𝟗) (F) 𝒙 = −𝟕 (H) 𝒙 = −𝟗

5. To get the graph of 𝒚 = 𝟑(𝒙 + 𝟏)𝟐 − 𝟓, we need two steps to shift the
original graph. The original graph is ________ (𝒚 = 𝒙𝟐 , 𝒚 = 𝟑𝒙𝟐 ).
Step 1: Shift the original graph to the _______ (left or right) ____ (1 or 5)
unit(s) to get the graph of 𝒚 = 𝟑(𝒙 + 𝟏)𝟐 .
Step 2: Further shift the graph of 𝒚 = 𝟑(𝒙 + 𝟏)𝟐 _______ (up or down) ____
(1 or 5) units to get the graph of 𝒚 = 𝟑(𝒙 + 𝟏)𝟐 − 𝟓.

6. Steve uses a different steps to get the graph of 𝒚 = 𝟑(𝒙 + 𝟏)𝟐 − 𝟓. First, he
shifts the graph of 𝒚 = 𝟑𝒙𝟐 down 5 units and get the graph of 𝒚 = 𝟑𝒙𝟐 − 𝟓.
Then he further shift the graph of 𝒚 = 𝟑𝒙𝟐 − 𝟓 to the left 1 unit to get the
final graph of 𝒚 = 𝟑(𝒙 + 𝟏)𝟐 − 𝟓. Do you think his method is correct? ____

7. The graph of 𝒚 = 𝟒(𝒙 − 𝟐)𝟐 + 𝟔 is ________ than the graph of 𝒚 = 𝒙𝟐 .


(A) wider (B) narrower

8. To get the graph of 𝒚 = 𝟒(𝒙 − 𝟐)𝟐 + 𝟔 , the graph of 𝒚 = 𝟒𝒙𝟐 is _______.


(A) moved to the left 2 units and up 6 units.
(B) moved to the left 2 units and down 6 units.
(C) moved to the right 2 units and up 6 units.
(D) moved to the right 2 units and down 6 units.

Note: We can either shift the graph up/down first or shift the graph left/right first,
we finally get the same graph. So the answer to 6 is correct.

29
8. Match each function with its graph.
𝟏 𝟏
(1) 𝒚 = (𝒙 + 𝟒)𝟐 − 𝟑 _____ (2) 𝒚 = (𝒙 − 𝟒)𝟐 + 𝟑 _____
𝟐 𝟐
𝟏 𝟏
(3) 𝒚 = (𝒙 − 𝟒)𝟐 − 𝟑 _____ (4) 𝒚 = (𝒙 + 𝟒)𝟐 + 𝟑 _____
𝟐 𝟐

A B

C D

9. If the vertex is (𝟖, 𝟏𝟎), then the function 𝒚 = (𝒙 − 𝒉)𝟐 + 𝒌 is


______________________, where 𝒂 =______.

10. If the vertex is (−𝟐, 𝟕) and 𝒂 = 𝟒, then the function 𝒚 = 𝒂(𝒙 − 𝒉)𝟐 + 𝒌
is ____________________.
11. If we shift the parabola of 𝒚 = (𝒙 + 𝟐)𝟐 − 𝟗 five units down, then we get
the parabola of the function _____________________ (In vertex form).

30
Practice 2

1. (1) We can translate the graph of 𝒚 = 𝟐𝒙𝟐 to the _________ 5 units to get
the graph of 𝒚 = 𝟐(𝒙 + 𝟓)𝟐 . (2) We can translate the graph of 𝒚 = 𝟔𝒙𝟐
_______ 9 units to get the graph of 𝒚 = 𝟔𝒙𝟐 + 𝟗.
(A) Up (B) Down (C) Left (D) Right

2. We can translate the parabola of ________________ to the right 2 units to


get the parabola of 𝒚 = 𝟕(𝒙 − 𝟐)𝟐 + 𝟗.
(A) 𝒚 = 𝒙𝟐 (B) 𝒚 = 𝒙𝟐 + 𝟗
(C) 𝒚 = 𝟕𝒙𝟐 (D) 𝒚 = 𝟕𝒙𝟐 + 𝟗

3. If we translate the graph of 𝒚 = (𝒙 − 𝟑)𝟐 − 𝟖 to the left 𝟑 units and then


translate the new graph 𝟑 units down, then we get the graph of ________.
(A) 𝒚 = (𝒙 − 𝟔)𝟐 − 𝟓 (B) 𝒚 = 𝒙𝟐 − 𝟓
(C) 𝒚 = (𝒙 − 𝟔)𝟐 − 𝟏𝟏 (D) 𝒚 = 𝒙𝟐 − 𝟏𝟏

4. Fill in the table.


Axis of Opening
Vertex (x, y)
symmetry direction
𝒚 = (𝒙 + 𝟐)𝟐 − 𝟏
𝒚 = 𝟑(𝒙 − 𝟕)𝟐 + 𝟐
𝟏
𝒚 = − (𝒙 − 𝟕)𝟐 + 𝟐
𝟒
𝒚 = −𝟗(𝒙 + 𝟓)𝟐 + 𝟖
𝟐
𝒚 = − (𝒙 − 𝟓 )𝟐 − 𝟖
𝟑
𝒚 = 𝒙𝟐 − 𝟖
𝟏
𝒚= (𝒙 + 𝟐)𝟐
𝟐

31
5. True or False.
𝟏
(1) The parabola of 𝒚 = 𝒙𝟐 − 𝟓 has the same opening direction with the
𝟒
parabola of 𝒚 = 𝟒𝒙𝟐 + 𝟓. _______
𝟏
(2) The parabola of 𝒚 = 𝒙𝟐 + 𝟓 has the same opening direction with the
𝟒
parabola of 𝒚 = 𝟒𝒙𝟐 . _______
(3) The graph of the function 𝒚 = −𝒙𝟐 has the same shape with the graph
of 𝒚 = 𝒙𝟐 + 𝟏. _______
𝟏
(4) The graphs of the functions 𝒚 = (𝒙 + 𝟏)𝟐 and 𝒚 = 𝟒(𝒙 + 𝟏)𝟐 have
𝟒
different vertex. _______
(5) The graphs of the functions 𝒚 = 𝟒(𝒙 − 𝟏)𝟐 and 𝒚 = −𝟒(𝒙 − 𝟏)𝟐 + 𝟏
have different axes of symmetry. _______
(6) If we shift the graph of 𝒚 = 𝟒𝒙𝟐 up 3 units along y-axis, then we get
the graph of 𝒚 = 𝟒𝒙𝟐 + 𝟑. _______

6. On the above graph, the vertex is ________. The function represented by


the graph could be ___________.
(A) (𝟒, 𝟑) (B) (−𝟒, 𝟑)
𝟏 𝟏
(C) 𝒚 = (𝒙 − 𝟒)𝟐 − 𝟑 (D) 𝒚 = (𝒙 + 𝟒)𝟐 + 𝟑
𝟐 𝟐
𝟏 𝟏
(E) 𝒚 = − (𝒙 − 𝟒)𝟐 + 𝟑 (F) 𝒚 = − (𝒙 + 𝟒)𝟐 − 𝟑
𝟐 𝟐

32
7. Fill in the blanks to complete the squares.
(1) 𝒂𝟐 ± 𝟐𝒂𝒃 + 𝒃𝟐 = (𝒂 ± ____ )𝟐

(2) 𝒙𝟐 + 𝟔𝒙 + ( )𝟐 = (𝒙 + ____ )𝟐

(3) 𝒙𝟐 − 𝟑𝒙 + ( )𝟐 = (𝒙 − _____ )𝟐
𝟐

(4) 𝒙𝟐 + 𝟐𝒙 + ( ) = ( _________ )𝟐

(5) 𝒙𝟐 − 𝟒𝒙 + ( ) = _________

(6) 𝒙𝟐 + 𝒃𝒙 + ( ) = (𝒙 + )𝟐

Think: (1) Why can we shift the parabola of 𝒚 = 𝒙𝟐 up two units to get the
parabola of 𝒚 = 𝒙𝟐 + 𝟐 ?

(2) Why can we shift the parabola of 𝒚 = 𝟐𝒙𝟐 to the left one unit
to get the parabola of 𝒚 = 𝟐(𝒙 + 𝟏)𝟐 ?

33
The Minimum and Maximum Value

If 𝒂 > 𝟎 in 𝒚 = 𝒂𝒙𝟐 + 𝒃𝒙 + 𝒄,
The parabola opens upward.
The vertex is the minimum point
or lowest point, of the parabola.

If 𝒂 < 𝟎 in 𝒚 = 𝒂𝒙𝟐 + 𝒃𝒙 + 𝒄,
The parabola opens downward.
The vertex is the maximum point
or highest point, of the parabola.

The minimum or maximum value of a function is the 𝒚-coordinate of the


vertex. If a quadratic in vertex form 𝒚 = 𝒂(𝒙 − 𝒉)𝟐 + 𝒌, then the vertex
is (𝒉, 𝒌). And 𝒌 is the minimum or maximum value of the function.

𝟏
1. Given the function 𝒚 = (𝒙 − 𝟐)𝟐 − 𝟔,
𝟑
(1) Since 𝒂 > 𝟎, it has a _______ value.
A) minimum B) maximum
(2) The minimum value is ______.
A) −𝟐 B) 𝟐
C) −𝟔 D) 𝟔

𝟏
2. Given the function 𝒚 = − (𝒙 − 𝟐)𝟐 − 𝟔,
𝟑
(1) Since 𝒂 < 𝟎, the vertex is a _______. A) minimum B) maximum
(2) The vertex of the function is ______. The maximum value is ______.
A) (𝟐, 𝟔) B) (−𝟐, 𝟔) E) −𝟐 G) 𝟐
C) (𝟐, −𝟔) D) (−𝟐, −𝟔) F) −𝟔 H) 𝟔

34
3. The function 𝒚 = 𝟑(𝒙 + 𝟏)𝟐 + 𝟓 has a ___________ ( minimum or maximum)
value, which is _______. That means all of the values of 𝒚 ≥ ______.
The vertex is _________. The axis of symmetry is ________.

Note: In the function 𝒚 = 𝟑(𝒙 + 𝟏)𝟐 + 𝟓, if (𝒙 + 𝟏)𝟐 = 𝟎, then 𝒚 = _____.


That means when 𝒙 = −𝟏, the function has the minimum. No matter what
value 𝒙 is, the y-value of the function is greater than or equal to 5.

4. The function 𝒚 = −𝟑(𝒙 + 𝟏)𝟐 + 𝟓 has a __________ (minimum or maximum)


value. When 𝒙 =______, the maximum value occurs, which is _______.
A) 𝟏 B) −𝟏 C) 𝟓 D) −𝟓

𝟐
5. The function 𝒚 = − (𝒙 − 𝟕)𝟐 − 𝟗 has a __________ (minimum or maximum)
𝟓
value. The maximum value occurs when 𝒙 is ______. And the range (all the
y-values) of the function is ________.

6. Find the minimum or maximum value and the range of a function.

A B C
It has a ___________ It has a ___________ It has a __________
value, that is _______. value, that is _______. value, that is ______.
The range is _______. The range is _______. The range is _______.

7. If the graph of a function opens upward and its vertex is (𝟏, −𝟐), then the
range of the function is _________.
(A) smaller than or equal to 𝟏 (B) smaller than 𝟏
(C) greater than or equal to −𝟐 (D) greater than −𝟐

35
The Function of 𝒚 = 𝒂𝒙𝟐 + 𝒃𝒙 + 𝒄

𝒚 = 𝒂𝒙𝟐 + 𝒃𝒙 + 𝒄 is the standard form of a quadratic function, where 𝒂 ≠ 𝟎 .


𝒚 = 𝒂(𝒙 − 𝒉)𝟐 + 𝒌 can be converted to the standard form. For example:

𝒚 = 𝟐(𝒙 − 𝟑)𝟐 + 𝟐
= 𝟐(𝒙𝟐 − 𝟔𝒙 + 𝟗) + 𝟐
= 𝟐𝒙𝟐 − 𝟏𝟐𝒙 + 𝟏𝟖 + 𝟐
= 𝟐𝒙𝟐 − 𝟏𝟐𝒙 + 𝟐𝟎

Given the function 𝒚 = 𝒂𝒙𝟐 + 𝒃𝒙 + 𝒄, we can find the vertex and the axis of
symmetry by converting it to its vertex form. We can convert the standard
form function to the vertex form using the process of completing the square.

Example 1: Convert the function 𝒚 = 𝟐𝒙𝟐 − 𝟏𝟐𝒙 + 𝟐𝟎 to its vertex form.


Solution: We have learned how to complete the square.

Step 1: Ignore the constant term. 𝒚 = 𝟐𝒙𝟐 − 𝟏𝟐𝒙 + 𝟐𝟎


Deal with the first two terms.
Step 2: Factor out the leading 𝒚 = 𝟐(𝒙𝟐 − 𝟔𝒙) + 𝟐𝟎
coefficient from the first two terms.

Step 3: Add two opposite numbers 𝒚 = 𝟐(𝒙𝟐 − 𝟔𝒙 + 𝟑𝟐 − 𝟑𝟐 ) + 𝟐𝟎


within the parenthesis. The absolute
value of the opposite numbers is the (The opposite numbers +9 −9 make 0.)
missing term to completing the square.
Step 4: Complete the square. 𝒚 = 𝟐[(𝒙 − 𝟑)𝟐 − 𝟗] + 𝟐𝟎

Step 5: Multiply out the brackets. 𝒚 = 𝟐(𝒙 − 𝟑)𝟐 − 𝟏𝟖 + 𝟐𝟎


Step 6: Simplify the expression
𝒚 = 𝟐(𝒙 − 𝟑)𝟐 + 𝟐
and get the vertex form.

36
Try 1: Convert the function 𝒚 = 𝟑𝒙𝟐 − 𝟔𝒙 − 𝟒 to the vertex form .
Solution: 𝒚 = 𝟑𝒙𝟐 − 𝟔𝒙 − 𝟒
𝒚 = 𝟑(𝒙𝟐 − ____𝒙) − 𝟒
𝒚 = 𝟑(𝒙𝟐 − ____𝒙 + (_____)𝟐 − (_____)𝟐 ) − 𝟒
𝒚 = 𝟑[(𝒙 − _____)𝟐 − _____] − 𝟒
𝒚 = 𝟑(𝒙 − _____)𝟐 − _____ − 𝟒
𝒚 = 𝟑(𝒙 − _____)𝟐 + (______)

Try 2: Please find the vertex and the minimum value of 𝒚 = 𝟒𝒙𝟐 + 𝟏𝟔𝒙 + 𝟐𝟐.
Hint: Convert it to the vertex form by completing the square first.

Solution: 𝒚 = 𝟒𝒙𝟐 + 𝟏𝟔𝒙 + 𝟐𝟐


𝒚 = (𝟒𝒙𝟐 + 𝟏𝟔𝒙) + 𝟐𝟐
𝒚=
𝒚=
𝒚=
𝒚=
𝒚 = 𝟒(𝒙 + ______)𝟐 + (______)
𝒚 = 𝟒(𝒙 − ______)𝟐 + (______)

Answer: The vertex is _____________. The minimum value is ___________.

Try 3: Convert the function 𝒚 = −𝟐𝒙𝟐 − 𝟏𝟔𝒙 − 𝟐𝟗 to the vertex form by


completing the square and find the axis of the symmetry.

Solution: 𝒚 = −𝟐𝒙𝟐 − 𝟏𝟔𝒙 − 𝟐𝟗


𝒚=

37
It is a long process to convert the standard form to the vertex form by the method
of completing the square. We may find a simple and convenient method to do it.
Example 2: Let’s convert the standard form 𝒚 = 𝒂𝒙𝟐 + 𝒃𝒙 + 𝒄 to its vertex form.
Solution:

Step 1: Ignore the constant term. 𝒚 = 𝒂𝒙𝟐 + 𝒃𝒙 + 𝒄


Deal with the first two terms.
𝒃
Step 2: Factor out the leading 𝒚 = 𝒂(𝒙𝟐 + 𝒙) + 𝒄
𝒂
coefficient from the first two terms.
Step 3: Add two opposite numbers 𝒚=
𝒃 𝒃 𝟐 𝒃 𝟐
within the parenthesis. The absolute 𝒂 (𝒙𝟐 + 𝒙 + ( ) − ( ) ) + 𝒄
value of the opposite numbers is the 𝒂 𝟐𝒂 𝟐𝒂

missing term to completing the square. (The two opposite numbers make 0.)

Step 4: Complete the square. 𝒃 𝟐 𝒃 𝟐


𝒚 = 𝒂[(𝒙 + ) −( ) ]+𝒄
𝟐𝒂 𝟐𝒂

Step 5: Multiply out the brackets. 𝒃 𝟐 𝒃𝟐


𝒚 = 𝒂(𝒙 + ) − +𝒄
𝟐𝒂 𝟒𝒂

Step 6: Simplify the expression 𝒃 𝟐 𝟒𝒂𝒄−𝒃𝟐


𝒚 = 𝒂(𝒙 + ) +
and get : 𝟐𝒂 𝟒𝒂

Step 7: We get the vertex form of a quadratic function:


𝒃 𝟒𝒂𝒄−𝒃𝟐
𝒚 = 𝒂[𝒙 − (− )]𝟐 +
𝟐𝒂 𝟒𝒂

𝟐 𝒃 𝟒𝒂𝒄−𝒃𝟐
So the vertex of 𝒚 = 𝒂𝒙 + 𝒃𝒙 + 𝒄 is (− , ). *************
𝟐𝒂 𝟒𝒂
𝒃 𝒃
− is the x-coordinate of the vertex and the axis of symmetry is 𝒙 = − .
𝟐𝒂 𝟐𝒂
𝟒𝒂𝒄−𝒃𝟐 𝒃
The minimum or maximum value is or the y-value when 𝒙 = − .
𝟒𝒂 𝟐𝒂

Using the above formulas, we can convert a quadratic function to its vertex
form without the process of completing the square. We can also directly find
the vertex, the axis of symmetry, the minimum or maximum value of a function.
38
Find the Vertex Using the Formula

Example 1: Convert 𝒚 = 𝟐𝒙𝟐 + 𝟒𝒙 + 𝟓 to its vertex form using the formulas.

Solution: In the function 𝒂 = 𝟐 , 𝒃 = 𝟒 , 𝒄 = 𝟓 .


𝒃 𝟒
Step 1: − =− = −𝟏. So the axis of symmetry is the line of 𝒙 = −𝟏.
𝟐𝒂 𝟐•𝟐
Step 2: Since 𝒂 > 𝟎, the vertex is a minimum, which occurs at 𝒙 = −𝟏.
When 𝒙 = −𝟏, the minimum 𝒚 = 𝟐(−𝟏)𝟐 + 𝟒(−𝟏) + 𝟓 = 𝟑.

Step 3: The vertex is (−𝟏, 𝟑) , so 𝒉 = −𝟏, 𝒌 = 𝟑.


Since 𝒂 = 𝟐 and the vertex form is 𝒚 = 𝒂(𝒙 − 𝒉)𝟐 + 𝒌 ,
𝒚 = 𝟐(𝒙 − (−𝟏))𝟐 + 𝟑 That is, 𝒚 = 𝟐(𝒙 + 𝟏)𝟐 + 𝟑.

Try 1: Convert 𝒚 = 𝟑𝒙𝟐 + 𝟔𝒙 + 𝟖 to its vertex form using the formula.


(0) In the function 𝒂 =____ , 𝒃 =_____ , 𝒄 =_____ .
𝒃 ( )
(1) − =− = _________, the axis of symmetry is 𝒙 =_______.
𝟐𝒂 𝟐( )

(2) The minimum value is 𝒚 = 𝟑( )𝟐 + 𝟔( ) + 𝟖 = _________.


(3) The vertex is ( , ).
(4) The vertex form of the function is: ______________________.

39
𝟏
Try 2: Convert 𝒚 = − 𝒙𝟐 + 𝟒𝒙 − 𝟑 to its vertex form using the formula.
𝟐
𝒃
(1) − =____________________, the axis of symmetry is _________.
𝟐𝒂
(2) The maximum value is _________. (3) The vertex is ___________.
(4) The vertex form of the function is:

The y-intercept is the point at which the parabola intercepts the y-axis. That
is the y-value when 𝒙 = 𝟎. Every quadratic function has only one y-intercept.

Example 2: What is the y-intercept


of the function 𝒚 = 𝟐𝒙𝟐 + 𝟒𝒙 + 𝟓?

Solution: To find the y-intercept,


substitute 0 for x and solve for y.

Let 𝒙 = 𝟎, then 𝒚 = 𝟐𝒙𝟐 + 𝟒𝒙 + 𝟓


= 𝟐(𝟎)𝟐 + 𝟒(𝟎) + 𝟓
=𝟓
Answer: The y-intercept is (0, 5).

Think: What is the y-intercept of the function 𝒚 = 𝒂𝒙𝟐 + 𝒃𝒙 + 𝒄 ?


Let 𝒙 = 𝟎, we get 𝒚 = 𝒄. So the y-intercept of a quadratic function is (𝟎, 𝒄 ).

Example 3: True or false?


(1) In the function 𝒚 = 𝟐𝒙𝟐 + 𝟒𝒙 + 𝟓, the constant term 𝟓 is the minimum value.
(2) In the function 𝒚 = 𝟐(𝒙 + 𝟏)𝟐 + 𝟑, the constant term 𝟑 is the minimum value.
(3) 𝒚 = 𝟐(𝒙 + 𝟏)𝟐 + 𝟑 is the vertex form of the function 𝒚 = 𝟐𝒙𝟐 + 𝟒𝒙 + 𝟓.

Solution: From example 1, we can see the statement (3) is true.


In statement (2) , the function is in the vertex form. Its constant term
is the y-coordinate of the vertex, which is the minimum value. So (2) is true.
In statement (1) , the function is in the standard form. Its constant term
is the y-intercept: when 𝒙 = 𝟎, 𝒚 = 𝟓, which is not the vertex. So (1) is False.

40
The x-intercept is the point at which the graph of the function intercepts the
x-axis. That is the x-value when 𝒚 = 𝟎. Some functions have no x-intercepts.
For example, the above function 𝒚 = 𝟐𝒙𝟐 + 𝟒𝒙 + 𝟓 has no x-intercepts.

Example 4: What is the x-intercepts


of the function 𝒚 = 𝒙𝟐 − 𝟔𝒙 + 𝟓?

Solution: To find the x-intercepts,


let y = 0 and solve the equation for x.

Let y = 𝟎, then 𝒙𝟐 − 𝟔𝒙 + 𝟓 = 𝟎
(𝒙 − 𝟏)(𝒙 − 𝟓) = 𝟎
𝒙𝟏 = 𝟏, 𝒙𝟐 = 𝟓
So the x-intercepts are (1, 0) and (5, 0).

Try 3: For the parabola of the function 𝒚 = −𝟐𝒙𝟐 + 𝟒𝒙 .


(1) The vertex is ___________.
(2) The y-intercept is the point _________.
(3) The x-intercepts are the points ___________.
(4) The vertex form of the function is:

Try 4: For the graph of the function 𝒚 = 𝟑𝒙𝟐 + 𝟔𝒙 − 𝟒 .


(1) The vertex is ___________.
(2) The y-intercept is the point _________.
(3) The x-intercepts are the points___________.
(4) The vertex form of the function is:

Try 5: Choice question. (A) The y-intercept (B) The y-value of the vertex
(1) In 𝒚 = −𝟐𝒙𝟐 − 𝟏𝟔𝒙 − 𝟐𝟗, the constant term −𝟐𝟗 is _____________.
(2) In 𝒚 = −𝟑(𝒙 + 𝟐)𝟐 − 𝟔, the constant term −𝟔 is ________________.

Try 6: The standard form of 𝒚 = −𝟑(𝒙 + 𝟐)𝟐 − 𝟔 is ________________.

41
Graphing Quadratic Functions

Example 1: How to graph 𝒚 = 𝟐𝒙𝟐 + 𝟒𝒙 + 𝟓 on the coordinate plane?

Hint: To graph a quadratic function,


𝒃
Step 1: Find the vertex first using − .
𝟐𝒂
Step 2: Set the vertex as the center point,
and find four more points that are
symmetric and close to the vertex.
Usually use five points to graph.
Sometimes the y-intercept helps.
Step 3: Plot the points and graph.

The following are the points we choose. The vertex is the center. The y-intercept.
Can you find the missing value 𝒙 −𝟑 −𝟐 −𝟏 𝟎 𝟏
according to the symmetry?
𝒚 𝟓 𝟑 𝟓 𝟏𝟏

𝒃 𝟒
Solution: Step 1: − =− = −𝟏 The vertex is (−𝟏, 𝟑).
𝟐𝒂 𝟐•𝟐
When 𝒙 = −𝟏, 𝒚 = 𝟐𝒙𝟐 + 𝟒𝒙 + 𝟓 = 𝟐(−𝟏)𝟐 + 𝟒(−𝟏) + 𝟓 = 𝟑.

Step 2: Find four more points close to the vertex.


When 𝒙 = 𝟎, 𝒚 = 𝟐(𝟎)𝟐 + 𝟒(𝟎) + 𝟓 = 𝟓.
When 𝒙 = 𝟏, 𝒚 = 𝟐( )𝟐 + 𝟒( ) + 𝟓 = 𝟏𝟏.
When 𝒙 = −𝟐, 𝒚 = 𝟐(−𝟐)𝟐 + 𝟒(−𝟐) + 𝟓 =_____.
When 𝒙 = −𝟑, 𝒚 = _________________________.

Step 3: The selected points to graph the function are in the above table.
Plot the five points and connect them with a smooth curve.
The above on the right is the graph of 𝒚 = 𝟐𝒙𝟐 + 𝟒𝒙 + 𝟓 .

42
When there are x-intercepts, the steps to graph quadratic functions is much easier.
𝒃
Step 1: Find the vertex first using − .
𝟐𝒂
Step 2: Find the y-intercept. Let 𝒙 = 𝟎 and find the y-value.
Step 3: Find the x-intercept(s). Solve the related equation for x.
Step 4: Graph the parabola using the points found in steps 1-3.

Example 2: Graph the function 𝒚 = −𝒙𝟐 + 𝟐𝒙 + 𝟑 on the coordinate plane.


𝒃 𝟐
Solution: Step 1: Find the vertex. − =− = 𝟏.
𝟐𝒂 𝟐 • (−𝟏)

When 𝒙 = 𝟏, 𝒚 = −(𝟏)𝟐 + 𝟐(𝟏) + 𝟑 = 𝟒. The vertex is (𝟏, 𝟒).

Step 2: When 𝒙 = 𝟎, 𝒚 = −𝒙𝟐 + 𝟐𝒙 + 𝟑 = −(𝟎)𝟐 + 𝟐(𝟎) + 𝟑 = 𝟑.


So the y-intercept is the point of (0, 3).

Step 3: When 𝒚 = 𝟎, −𝒙𝟐 + 𝟐𝒙 + 𝟑 = 𝟎


−(𝒙 + 𝟏)(𝒙 − 𝟑) = 𝟎
𝒙 = −𝟏, 𝒙 = 𝟑
So the x-intercepts are the points of (−1, 0) and (3, 0).

Step 4: The points to graph the function are in the following table.
Below on the right is the graph of 𝒚 = −𝒙𝟐 + 𝟐𝒙 + 𝟑.

𝒙 𝒚
−𝟏 𝟎

𝟎 𝟑 What is the
missing value
𝟏 𝟒 according to the
symmetry?
𝟐

𝟑 𝟎

43
𝟏
Try 1: Please graph 𝒚 = − 𝒙𝟐 + 𝟒𝒙 − 𝟑 on the coordinate plane.
𝟐
Hint: We do not need convert it to the vertex form to graph.

𝒙 Please fill
in the table.
𝒚

44
Try 2: Please graph 𝒚 = −(𝒙 − 𝟐)𝟐 + 𝟕 on the coordinate plane.

𝒙 𝒚

45
Try 3: (1) What is the vertex of the function 𝒚 = 𝒙𝟐 + 𝟗 ? _________
(2) What is the vertex of the function given in the following table?
(3) The y-intercept of the function in (2) is __________.
𝒙 −𝟑 −𝟐 −𝟏 𝟎 𝟏

𝒚 𝟏𝟖 𝟔 𝟐 𝟔 𝟏𝟖

Hint: Find the pairs of symmetric 𝒚-values first. The y-coordinate of the vertex
is the non-symmetric 𝒚-value in the table. 𝒙-values in the table may be not
symmetric.

Try 4: (1) What is the vertex of the function 𝒚 = 𝟐𝒙𝟐 + 𝟖𝒙 + 𝟏𝟏? _________

𝒙 −𝟐 −𝟏 𝟎 𝟏 𝟐

𝒚 −𝟏 −𝟒 −𝟓 −𝟒 −𝟏

(2) What is the vertex of the quadratic function given in the above table?
(3) The graph of the above function opens _________. (up, down)
(4) The axis of symmetry of the graph is __________.
(5) The minimum value of the function is __________.
(6) The y-intercept of the parabola is __________.

Try 5: (1) What is the vertex of the quadratic function given in the table?

𝒙 −𝟐 −𝟏 𝟎 𝟏 𝟐 𝟑

𝒚 −𝟐𝟒 −𝟗 𝟎 𝟑 𝟎 −𝟗

(2) The parabola of the above function opens _________. (up, down)
(3) The axis of symmetry of the parabola is __________.
(4) The maximum value of the function is __________.

Try 6: What is the vertex of the function given in the following table?

𝒙 ••• −𝟏 𝟎 𝟏 𝟐 •••

𝒚 ••• −𝟓 𝟏 𝟑 𝟏 •••

46
Practice 3

1. On the right is a graph of the


function 𝒚 = 𝒂𝒙𝟐 + 𝒃𝒙 + 𝒄.
In the function,
when x = 𝟎, y = _____ ;
when y = 𝟎, x = _____ .

a _____ 0. ( > , < )


c = _____.

2. (1) For the graph of 𝒚 = 𝟑𝒙𝟐 + 𝟏𝟐, the y-intercept is _________,


the x-intercepts are _______________.
(2) For the graph of 𝒚 = 𝟒𝒙𝟐 + 𝟑𝟐𝒙, the y-intercept is _________,
the x-intercepts are _______________.
𝟏 𝟏
(3) For the graph of 𝒚 = 𝒙𝟐 − 𝒙 − 𝟑, the y-intercept is _________,
𝟐 𝟐
the x-intercepts are _______________.
(4) The y-intercept of a quadratic function 𝒚 = 𝒂𝒙𝟐 + 𝒃𝒙 + 𝒄 always
occurs at (0, c). True or false? _________.
𝟏
(5) If the graph of 𝒚 = −𝒙𝟐 + 𝟐𝒙 + 𝒄 goes through the point of (0, ),
𝟐
then 𝒄 = _____.

𝟏
3. (1) Please rewrite 𝒚 = (𝒙 − 𝟒)𝟐 + 𝟔 in standard form.
𝟖
(2) The range is ________. The domain of the function is __________.

47
4. Please convert 𝒚 = 𝟓 − 𝟒𝒙 − 𝒙𝟐 to the vertex form using the formula.

5. Please rewrite 𝒚 = 𝒙𝟐 + 𝟑𝒙 − 𝟔 in vertex form by completing the square.

6. If we translate the graph of 𝒚 = −𝟐𝒙𝟐 − 𝟒𝒙 + 𝟏 to the right 𝟑 units and


then translate the new graph 𝟓 units up, then what will we get? ________

𝟏
7. (1) The constant term 6 of 𝒚 = 𝒙𝟐 − 𝟒𝒙 + 𝟔 is ____________.
𝟖
𝟏
(2) The constant term 6 of 𝒚 = (𝒙 − 𝟒)𝟐 + 𝟔 is ___________.
𝟖
(A) The y-intercept (B) The y-coordinate of the vertex
𝟏
(3) The y-intercept of 𝒚 = (𝒙 − 𝟒)𝟐 + 𝟔 is ____________.
𝟖
𝟏
(4) The minimum value of 𝒚 = (𝒙 − 𝟒)𝟐 + 𝟔 is ________.
𝟖

48
8. When completing the square for the function 𝒚 = 𝒙𝟐 − 𝟏𝟔𝒙 + 𝟗, in
order to find the minimum, we need write the function in vertex form
𝒚 = 𝒂(𝒙 − 𝒉)𝟐 + 𝒌 . What is the value of 𝒉 in the function? _________
(A) 𝟒 (B) −𝟒 (C) 𝟖 (D) −𝟖

9. The function 𝒚 = 𝟒𝒙𝟐 − 𝟏𝟔𝒙 + 𝟗 can be written in vertex form as _____.


(A) 𝒚 = (𝟒𝒙 − 𝟖)𝟐 − 𝟏𝟓 (C) 𝒚 = 𝟒(𝒙 − 𝟐)𝟐 − 𝟕
(B) 𝒚 = 𝟒(𝒙 − 𝟖)𝟐 + 𝟏𝟓 (D) 𝒚 = 𝟒(𝒙 − 𝟐)𝟐 + 𝟕

10. Which of the following function


could be represented by the graph
shown on the right? ________
𝟏
(A) 𝒚 = (𝒙 − 𝟔)𝟐 − 𝟏𝟎
𝟑
𝟏
(B) 𝒚 = (𝒙 + 𝟔)𝟐 + 𝟏𝟎
𝟑
𝟏
(C) 𝒚 = − (𝒙 − 𝟔)𝟐 + 𝟏𝟎
𝟑
𝟏
(D) 𝒚 = − (𝒙 + 𝟔)𝟐 − 𝟏𝟎
𝟑

11. If the graph of a function opens downward and its vertex is (𝟓, −𝟑),
then the y-values of the function is _________.
(A) greater than or equal to −𝟑 (B) greater than 𝟑
(C) smaller than or equal to −𝟑 (D) smaller than 𝟓

12. The axis of symmetry of the function 𝒚 = (𝒙 − 𝟏)𝟐 + 𝟑 is _________.


(A) 𝒙 = 𝟏 (B) 𝒙 = −𝟏
(C) 𝒙 = 𝟑 (D) 𝒙 = −𝟑

13. The axis of symmetry of the function 𝒚 = 𝒙𝟐 + 𝟔𝒙 is _________.


(A) 𝒙 = 𝟔 (B) 𝒙=𝟑
(C) 𝒙 = −𝟔 (D) 𝒙 = −𝟑

49
14. Which of the following quadratic function has the least minimum? _______
(A) 𝒚 = 𝟑𝒙𝟐 + 𝟔𝒙 − 𝟒 (B) 𝒚 = 𝟒(𝒙 − 𝟖)𝟐 + 𝟓

𝒙 𝒚
𝟏 𝟏𝟑

𝟎 𝟕

−𝟏 𝟓

−𝟐 𝟕

−𝟑 𝟏𝟑

(C) ( D)

15. Which of the following could be the graph of 𝒚 = 𝟑𝒙(𝟐 − 𝒙) ? ________

A B C

𝟏
16. The function 𝒚 = 𝒙𝟐 + 𝟐𝒙 + 𝟏 can be written in vertex form as _______.
𝟐
𝟏 𝟏 𝟏
(A) 𝒚 = (𝒙 − 𝟏)𝟐 + 𝟐 (C) 𝒚 = (𝒙 − 𝟏)𝟐 +
𝟐 𝟐 𝟐
𝟏 𝟏
(B) 𝒚 = (𝒙 − 𝟏)𝟐 − 𝟑 (D) 𝒚 = (𝒙 + 𝟐)𝟐 − 𝟏
𝟐 𝟐

50
17. Given the function 𝒚 = −𝒙𝟐 + 𝟐𝒙 + 𝟖, state whether the vertex represents
a maximum or minimum for the function. Explain your answer.

𝟒𝒂𝒄−𝒃𝟐
18. Given the minimum or maximum value of a quadratic function is ,
𝟒𝒂
find the maximum or minimum value of the function 𝒚 = 𝟑𝒙𝟐 + 𝟔𝒙 − 𝟒 .

𝟒𝒂𝒄−𝒃𝟐
19. Given the minimum or maximum value of a quadratic function is ,
𝟒𝒂
find the maximum or minimum value of the function 𝒚 = −𝟐𝒙𝟐 − 𝟒𝒙 + 𝟏 .

20. The graph representing a function is shown below. Which function has a
maximum that is greater than the
one shown in the graph? ______

(A) 𝒚 = −𝒙𝟐 + 𝟐𝒙 + 𝟑

(B) 𝒚 = 𝒙𝟐 + 𝟐𝒙 + 𝟑
𝟏
(C) 𝒚 = − 𝒙𝟐 − 𝟐𝒙 + 𝟒
𝟐
𝟏
(D) 𝒚 = 𝒙𝟐 + 𝟐𝒙 + 𝟒
𝟐

51
Quadratic Functions
and Quadratic Equations

𝒚 = 𝒂𝒙𝟐 + 𝒃𝒙 + 𝒄 is a quadratic function and could be an equation.


𝟎 = 𝒂𝒙𝟐 + 𝒃𝒙 + 𝒄 is only a quadratic equation. (where a ≠ 𝟎 ).
A quadratic equation is a special case of the quadratic function, when 𝒚 = 𝟎.
A quadratic function contains many different quadratic equations when 𝒚 are
different values. For example: For the function 𝒚 = 𝒙𝟐 − 𝟔𝒙 + 𝟓,
when 𝒚 = 𝟎, the function will be the equation 𝒙𝟐 − 𝟔𝒙 + 𝟓 = 𝟎 ;
when 𝒚 = 𝟏, the function will be the equation 𝒙𝟐 − 𝟔𝒙 + 𝟓 = 𝟏 ;
when 𝒚 = 𝟐, the function will be the equation 𝒙𝟐 − 𝟔𝒙 + 𝟓 = 𝟐 ;

To show the difference between functions and equations, we use a function


notation to mark functions as f(x). For Example: We can rewrite the above
function as f(x) = 𝒙𝟐 − 𝟔𝒙 + 𝟓. Here, f(𝒙) = 𝒚 .

Function Notation

𝒇(𝒙) = 𝒙𝟐 − 𝟔𝒙 + 𝟓

𝒇 is the name This tells you This tells you what


of the function the value of 𝒙 The function does

When 𝒙 = 𝟎, it will be 𝒇(𝟎) = 𝒙𝟐 − 𝟔𝒙 + 𝟓 = 𝟎𝟐 − 𝟔 • 𝟎 + 𝟓 = 𝟓


When 𝒙 = 𝟏, it will be 𝒇(𝟏) = 𝒙𝟐 − 𝟔𝒙 + 𝟓 = 𝟏𝟐 − 𝟔 • 𝟏 + 𝟓 = 𝟎
When 𝒙 = 𝟐, it will be 𝒇(𝟐) = 𝒙𝟐 − 𝟔𝒙 + 𝟓 = 𝟐𝟐 − 𝟔 • 𝟐 + 𝟓 = −𝟑

Given there are two different functions, we can mark them with different names
such as g(x) , h(x) , j(x). For example: g(x) = −𝟐𝒙𝟐 − 𝟕, h(x) = 𝒙 + 𝟒.

52
The zero of a function is the x-value that makes the function equal to 0.
To find the zero of a function means to solve the equation 𝒇(𝒙) = 𝟎 for x.
Example 1: Find the zero of the function 𝒇(𝒙) = 𝒙𝟐 − 𝟔𝒙 + 𝟓.

Solution: 𝒇(𝒙) = 𝒙𝟐 − 𝟔𝒙 + 𝟓 = 𝟎
(𝒙 − 𝟏)(𝒙 − 𝟓) = 𝟎 (In factored form)
𝒙 − 𝟏 = 𝟎 or 𝒙−𝟓=𝟎
Roots: 𝒙𝟏 = 𝟏 , 𝒙𝟐 = 𝟓

We can also find the zero of a function


from the graph of the function, which is
the x-values when 𝒚 = 𝟎. That is also
the x-coordinates of the x-intercepts.

On the right is the graph of the function


𝒇(𝒙) = 𝒙𝟐 − 𝟔𝒙 + 𝟓. We can see that
the x-intercepts are (1, 0) and (5, 0).
So 𝒙 = 1, 5 makes the function be 0.

Try 1: (1) Find the zero of the function 𝒇(𝒙) = 𝒙𝟐 − 𝟖𝒙 − 𝟗, algebraically.

(2) Given the function 𝒇(𝒙) = 𝒙𝟐 − 𝟔𝒙 + 𝟓, what is the value of 𝒇(𝟕)?


Hint: Find the value of the function, when 𝒙 = 𝟕.

53
Example 2: Given the function 𝒇(𝒙) = 𝒙𝟐 − 𝟔𝒙 + 𝟓 ,
find the function 𝒈(𝒙) = 𝒇(𝒙 + 𝟐).

Solution: 𝒈(𝒙) = 𝒇(𝒙 + 𝟐) (Where 𝒇(𝒙) = 𝒙𝟐 − 𝟔𝒙 + 𝟓)


= (𝒙 + 𝟐)𝟐 − 𝟔(𝒙 + 𝟐) + 𝟓 (Substitute 𝒙 + 𝟐 for )
= (𝒙𝟐 + 𝟒𝒙 + 𝟒) + (−𝟔𝒙 − 𝟏𝟐) + 𝟓 (Expand each term)
= 𝒙𝟐 + (𝟒𝒙 − 𝟔𝒙) + (𝟒 − 𝟏𝟐 + 𝟓) (Combine like terms)
= 𝒙𝟐 − 𝟐𝒙 − 𝟑 (Simplify)

Answer: The function 𝒈(𝒙) = 𝒙𝟐 − 𝟐𝒙 − 𝟑 .

Try 2: Given the function 𝒇(𝒙) = 𝒙𝟐 − 𝟐𝒙 + 𝟑 , find the following functions.


(1) 𝒈(𝒙) = 𝒇(𝒙 + 𝟓) (2) 𝒉(𝒙) = 𝒇(𝟒𝒙)

(3) 𝒋(𝒙) = 𝒇(𝒙) + 𝟒 (4) 𝒌(𝒙) = 𝟐[𝒇(𝒙)]

Try 3: Find the zero of the function 𝒇(𝒙) = 𝟒𝒙𝟐 − 𝟕𝒙 + 𝟑 , algebraically.

54
Try 4: If we shift the graph of 𝒇(𝒙) = (𝒙 − 𝟑)𝟐 − 𝟒 three units to the left, then
we get the graph of the function _____________________ (In vertex form).

Try 5: Given the function 𝒇(𝒙) = (𝒙 − 𝟑)𝟐 − 𝟒 ,


(1) find the coordinates of the vertex of the above function;
(2) find the new function defined by 𝒈(𝒙) = 𝒇(𝒙 + 𝟐) in vertex form;
(3) find the vertex of the function defined by 𝒈(𝒙) = 𝒇(𝒙 + 𝟐).

Try 6: Given the vertex of the function 𝒇(𝒙) has the coordinates (𝟑, −𝟒),
find the coordinates of the vertex of the function 𝒈(𝒙) = 𝒇(𝒙 + 𝟐).

Try 7: Given the function 𝒇(𝒙) = 𝒙𝟐 − 𝟔𝒙 + 𝟓 , find the coordinates of


the vertex of the parabola defined by 𝒈(𝒙) = 𝒇(𝒙 + 𝟑).

55
Solve Quadratic Equations by Graphing

Example 1: Solve the quadratic equation 𝒙𝟐 − 𝟐𝒙 − 𝟑 = 𝟎 by graphing.

Method 1: Since a quadratic equation is a special case of the related quadratic


function. We can solve the equation by graphing the related quadratic function
and find the roots on the graph. The roots are the x-coordinates when 𝒚 = 𝟎.

On the right is the graph of the


related function 𝒚 = 𝒙𝟐 − 𝟐𝒙 − 𝟑.

From the graph, we can see that


the function has two intercepts
with x-axis, which are ( , )
and ( , ). So when 𝒚 = 𝟎,
the equation have two real
roots, 𝒙𝟏 = −𝟏 , 𝒙𝟐 = 𝟑.

We can check it by solving the equation using factoring or the quadratic formula.
Method 2: The original equation 𝒙𝟐 − 𝟐𝒙 − 𝟑 = 𝟎
Factoring (𝒙 + 𝟏)(𝒙 − 𝟑) = 𝟎
𝒙 + 𝟏 = 𝟎 or 𝒙 − 𝟑 = 𝟎
Roots: 𝒙𝟏 = −𝟏 , 𝒙𝟐 = 𝟑 (Proved)

Try 1: (1) The above equation 𝒙𝟐 − 𝟐𝒙 − 𝟑 = 𝟎 has two roots and its
discriminant 𝒃𝟐 − 𝟒𝒂𝒄 _____ 𝟎. (A) < (B) > (C) =

(2) If the quadratic equation 𝒂𝒙𝟐 + 𝒃𝒙 + 𝒄 = 𝟎 (𝒂 ≠ 𝟎) has two roots:


𝒙𝟏 = −𝟏, 𝒙𝟐 = 𝟑, then the graph of the related function has ________
x-intercepts. The coordinates of x-intercepts are ( , ) and ( , ).
(A) 1 (B) 2 (C) 𝟎
56
Example 2: Solve the equation 𝒙𝟐 − 𝟔𝒙 + 𝟗 = 𝟎 by graphing.

Method 1: Solve the equation by graphing the related quadratic function. And
find the x-intercepts when 𝒚 = 𝟎. Below on the right is the graph of the related
function 𝒇(𝒙) = 𝒙𝟐 − 𝟔𝒙 + 𝟗.

From the graph, we can see that


the function has one x-intercept
which is the vertex ( , ).
So when 𝒚 = 𝟎, the equation
has one real root, 𝒙 =____.

We can check it by solving the


quadratic equation as follows.

Method 2: 𝒙𝟐 − 𝟔𝒙 + 𝟗 = 𝟎 Its discriminant 𝒃𝟐 − 𝟒𝒂𝒄 ____ 𝟎.


(𝒙 − 𝟑)𝟐 = 𝟎
Roots: 𝒙=𝟑 (Proved)

Example 3: Solve the equation 𝒙𝟐 − 𝟔𝒙 + 𝟏𝟏 = 𝟎 by graphing.

Method 1: Solve the equation by graphing. Find the x-intercepts when 𝒚 = 𝟎.


Below on the right is the graph of the related function 𝒇(𝒙) = 𝒙𝟐 − 𝟔𝒙 + 𝟏𝟏 .

From the graph, we can see that


the function has no intercept
with x-axis. So the equation
has no real root.

Method 2: In the original equation


𝒂 = 𝟏 , 𝒃 = −𝟔 , 𝒄 = 𝟏𝟏
𝒃𝟐 − 𝟒𝒂𝒄 = (−𝟔)𝟐 − 𝟒 ∙ 𝟏 ∙ 𝟏𝟏
= − 𝟖 < 𝟎. So it has ____ real roots.

57
Try 2: (1) Below is the graph of the function 𝒇(𝒙) = 𝟐𝒙𝟐 + 𝟒𝒙 − 𝟓 .

What are the roots of the


equation 𝟐𝒙𝟐 + 𝟒𝒙 − 𝟓 = 𝟎 ?

x-intercepts: ______________
Roots: __________________.
𝒃𝟐 − 𝟒𝒂𝒄 ____ 𝟎. ( > , = , < )

(2) Solve the equation −𝒙𝟐 + 𝟑𝒙 − 𝟕 = 𝟎 by graphing.

Given the graph of the related


function 𝒚 = −𝒙𝟐 + 𝟑𝒙 − 𝟕
on the right, please find

x-intercepts: ______________
Roots: __________________.
𝒃𝟐 − 𝟒𝒂𝒄 ____ 𝟎. ( > , = , < )

(3) Find the zero of the function 𝒇(𝒙) = 𝒙𝟐 − 𝟖𝒙 + 𝟏𝟔 by graphing.

Given the graph of the related


function 𝒚 = 𝒙𝟐 − 𝟖𝒙 + 𝟏𝟔
on the right, please find

x-intercepts: ______________
Roots: __________________.
𝒃𝟐 − 𝟒𝒂𝒄 ____ 𝟎. ( > , = , < )

58
Equation 𝒙𝟐 − 𝟔𝒙 + 𝟓 = 𝟎 𝒙𝟐 − 𝟔𝒙 + 𝟗 = 𝟎 𝒙𝟐 − 𝟔𝒙 + 𝟏𝟏 = 𝟎

Discriminant 𝒃𝟐 − 𝟒𝒂𝒄 > 𝟎 𝒃𝟐 − 𝟒𝒂𝒄 = 𝟎 𝒃𝟐 − 𝟒𝒂𝒄 < 𝟎

Graph of
Related
Function

Intercepts 2 x-intercepts 1 x-intercept 0 x-intercepts

Real Roots Two Roots One Root None

Try 3: (1) If the quadratic function 𝒚 = 𝒂𝒙𝟐 + 𝒃𝒙 + 𝒄 has real roots,


then the zeros of the function must occurs at ______.
(A) 𝒙 = 𝟎 (B) 𝒙 = 𝟏 (C) 𝒚=𝟎 (D) 𝒚 = 𝟏

(2) When the graph of the function 𝒚 = 𝒂𝒙𝟐 + 𝒃𝒙 + 𝒄 has only one intercept
with x-axis, the equation 𝒂𝒙𝟐 + 𝒃𝒙 + 𝒄 = 𝟎 has _______ real root(s) and its
discriminant 𝒃𝟐 − 𝟒𝒂𝒄 _____ 𝟎.
(A) 𝟏 (B) 𝟐 (C) < (D) = (E) >

(3) When 𝒃𝟐 − 𝟒𝒂𝒄 _____ 𝟎, the graph of the function 𝒚 = 𝒂𝒙𝟐 + 𝒃𝒙 + 𝒄 has
no x-intercepts. And the equation 𝒂𝒙𝟐 + 𝒃𝒙 + 𝒄 = 𝟎 has ______ real root(s).
(A) 𝟏 (B) 𝟎 (C) < (D) = (E) >

(4) The vertex of the graph of the function 𝒚 = 𝒂𝒙𝟐 + 𝒃𝒙 + 𝒄 is above the
x-axis. If 𝒂 > 𝟎, then _____________; if 𝒂 < 𝟎, then _____________.
(A) 𝒃𝟐 − 𝟒𝒂𝒄 ≥ 𝟎 (B) 𝒃𝟐 − 𝟒𝒂𝒄 < 𝟎
(C) 𝒃𝟐 − 𝟒𝒂𝒄 > 𝟎 (D) 𝒃𝟐 − 𝟒𝒂𝒄 = 𝟎

59
Try 4: Below are the graphs of the quadratic functions.
How many real roots do the related equations have?

(A) __________ (B) __________

(C) __________ (D) __________

Try 5: Below is a table of the quadratic function.


(1) The vertex of the function is ________ .
(2) How many real roots does the related equation have? _______
(3) Please find the zero of the function. _______

𝒙 𝟎 𝟏 𝟐 𝟑 𝟒 𝟓
𝒚 𝟓 𝟎 −𝟑 −𝟒 −𝟑 𝟎

60
Example 4: Below is the graph of the function 𝒚 = 𝒙𝟐 − 𝟐𝒙 − 𝟖.
Find: (1) The x-intercepts of the function are __________ and __________.
(2) When x = ______, the value of the function 𝒚 = 𝟎 ?
(3) When x = ______, the value of the function 𝒚 < 𝟎 ?
(4) When x = ______, the value of the function 𝒚 > 𝟎 ?

Solution: When the points on the parabola is above the x-axis, 𝒚 > 𝟎 .
When the points on the parabola is on the x-axis, 𝒚 = 𝟎 .
When the points on the parabola is below the x-axis, 𝒚 < 𝟎 .

(1) The x-intercepts of the function


are (𝟎, −𝟐) and (𝟎, 𝟒) .

(2) When 𝒙 = −𝟐 and 𝒙 = 𝟒,


the value of the function 𝒚 = 𝟎.

(3) When −𝟐 < 𝒙 < 𝟒,


the value of the function 𝒚 < 𝟎.

(4) When 𝒙 < −𝟐 and 𝒙 > 𝟒,


the value of the function 𝒚 < 𝟎.

Try 6: On the right is the graph


of the quadratic function
𝒚 = 𝒂𝒙𝟐 + 𝒃𝒙 + 𝒄 .

(1) The x-intercepts of the function


are __________ and __________.
(2) When 𝒙 = _________, 𝒚=𝟎.
(3) When 𝒙____________, 𝒚>𝟎.
(4) When 𝒙 ____________, 𝒚 < 𝟎 .

61
Example 5: Solve the equation −𝟐𝒙𝟐 − 𝟒𝒙 + 𝟑 = 𝟎 by graphing.

Solution: The roots of quadratic equations are usually not integers. Sometimes
we need estimate the approximate roots of the equation. In this case, if integer
roots can not be found, estimate the roots to the nearest tenth.

Graph the related function


𝒇(𝒙) = −𝟐𝒙𝟐 − 𝟒𝒙 + 𝟑
The x-intercepts are located between
𝟎 and 𝟏 and between −𝟑 and −𝟐.

Make a table using an increment of


0.1 for the x-values located between
𝟎 and 𝟏 and between −𝟑 and −𝟐.

Use the mean of the interval to narrow it.


Use a calculator to find each y-value. Then look for a change in the signs of the
function values. The function value that close to zero is the best approximation
for a zero of the function.
𝒙 𝟎. 𝟑 𝟎. 𝟒 𝟎. 𝟓 𝟎. 𝟔 𝟎. 𝟕 between
𝒚 𝟏. 𝟔𝟐 𝟏. 𝟎𝟖 𝟎. 𝟓 −𝟎. 𝟏𝟐 −𝟎. 𝟕𝟖 𝟎 and 𝟏

𝒙 −𝟐. 𝟕 −𝟐. 𝟔 −𝟐. 𝟓 −𝟐. 𝟒 −𝟐. 𝟑 between


𝒚 −𝟎. 𝟕𝟖 −𝟎. 𝟏𝟐 𝟎. 𝟓 𝟏. 𝟎𝟖 𝟏. 𝟔𝟐 −𝟑 and −𝟐

For each table, there are two y-values close to zero when the sign changes, which
are – 𝟎. 𝟏𝟐 and 𝟎. 𝟓. But – 𝟎. 𝟏𝟐 is more close to zero than 𝟎. 𝟓, so we choose
the x-values related to – 𝟎. 𝟏𝟐. Thus, the roots are approximately – 𝟐. 𝟔 and 𝟎. 𝟔.

Try 7: Below is the table of values of the quadratic function 𝒚 = 𝒂𝒙𝟐 + 𝒃𝒙 + 𝒄.


The values are close to the x-intercept. Then the root of 𝒂𝒙𝟐 + 𝒃𝒙 + 𝒄 = 𝟎
is 𝒙 = _____. 𝒙 𝟓. 𝟒 𝟓. 𝟓 𝟓. 𝟔 𝟓. 𝟕

𝒚 −𝟎. 𝟖𝟒 −𝟎. 𝟐𝟓 𝟎. 𝟑𝟔 𝟎. 𝟗𝟗

62
The Axis of Symmetry

The axis of symmetry is the vertical line that goes through the vertex
𝒃
of a quadratic function. We have learned it is the line of 𝒙 = − .
𝟐𝒂

If a quadratic equation has two roots 𝒙𝟏 and 𝒙𝟐 , that means the graph of the
related function has two x-intercepts, then the axis of symmetry is halfway
between the two x-intercepts. That is, the axis of symmetry of the related
𝟏
function is the line of 𝒙 = (𝒙𝟏 + 𝒙𝟐 ) .
𝟐

Example 1: A quadratic equation


𝒙𝟐 − 𝟔𝒙 + 𝟓 = 𝟎 has two roots 𝒙𝟏 = 𝟏,
and 𝒙𝟐 = 𝟓. Find the axis of symmetry
of the related function.

On the right is the graph of the related


function 𝒇(𝒙) = 𝒙𝟐 − 𝟔𝒙 + 𝟓. It has
two x-intercepts: (1, 0) and (5, 0).

𝟏 𝟏 𝟔
Solution: 𝒙 = (𝒙𝟏 + 𝒙𝟐 ) = (𝟏 + 𝟓) = = 𝟑
𝟐 𝟐 𝟐

Answer: The axis of symmetry is the line of 𝒙 = 𝟑.


𝒃
We can prove it. Since the axis of symmetry is the line of = − ,
𝟐𝒂
−𝟔
the axis of symmetry of the above function is 𝒙 = − = 𝟑. (Proved)
𝟐•𝟏

Try 1: The quadratic equation 𝒙𝟐 − 𝟐𝒙 − 𝟑 = 𝟎 has two roots 𝒙𝟏 = −𝟏,


and 𝒙𝟐 = 𝟑. Find the axis of symmetry of the related function. See the
graph on the next page. Please draw the axis of symmetry on the graph.

63
Try 2: The quadratic equation −𝟐𝒙𝟐 + 𝟒𝒙 = 𝟎 has two roots 𝒙𝟏 = 𝟎,
and 𝒙𝟐 = 𝟐. Find the axis of symmetry of the related function. See the
above graph on the right. Please draw the axis of symmetry on the graph.

The axis of symmetry of a quadratic function is halfway between any pair of


two symmetric points of the function. If p and q represents the x-coordinates
𝟏
of two symmetric points, then the axis of symmetry is 𝒙 = (𝒑 + 𝒒) .
𝟐

Example 2: The parabola of the quadratic function 𝒇(𝒙) = −𝟐𝒙𝟐 + 𝟒𝒙


𝟏 𝟏 𝟏 𝟏
goes through two points: (− , −𝟐 ) and ( 𝟐 , −𝟐 ). Find the axis
𝟐 𝟐 𝟐 𝟐
𝟏 𝟏
of symmetry of the function. Hint: Here, 𝒑 = − , 𝒒 = 𝟐 .
𝟐 𝟐

𝟏 𝟏 𝟏 𝟏 𝟐
Solution: 𝒙 = (𝒑 + 𝒒) = (− + 𝟐 ) = = 𝟏
𝟐 𝟐 𝟐 𝟐 𝟐

Answer: The axis of symmetry is the line of 𝒙 = 𝟏.

Try 3: The parabola of the quadratic function 𝒇(𝒙) = 𝒙𝟐 − 𝟐𝒙 − 𝟑 goes


through two symmetric points: (−𝟐 , 𝟓 ) and (𝟒 , 𝟓 ). Find the axis of
symmetry of the function. See the above graph on the left.
64
Try 4: Given the points on the graph
of a certain quadratic function shown
on the right, find the coordinates of
the symmetric points and mark them
on the graph.
(1) (𝟔 , 𝟓 ) and _________.
(2) (𝟕 , 𝟎 ) and _________.
(3) (𝟓 , 𝟖 ) and _________.
(4) (𝟏 , −𝟕 ) and _________.

Try 5: The parabola of the quadratic function 𝒚 = 𝒂𝒙𝟐 + 𝒃𝒙 + 𝒄 (𝒂 ≠ 𝟎) has


two intercepts with x-axis, which are (−𝟑 , 𝟎 ) and (𝟕 , 𝟎). What is the axis
of symmetry of the function?

Try 6: The parabola of the quadratic function has two x-intercepts and the axis
of symmetry 𝒙 = 𝟒. One x-intercept is (𝟏 , 𝟎 ), the other x-intercept is _______.
(A) (𝟓, 𝟎) (B) ( 𝟔, 𝟎)
(C) (𝟕, 𝟎) (D) (𝟖, 𝟎)

Try 7: The parabola of the quadratic function has the axis of symmetry 𝒙 = 𝟔.
If the parabola goes through the point of (𝟑 , 𝟕 ), then the parabola goes through
the symmetric point: ________.
(A) (𝟔, 𝟎) (B) ( 𝟔, 𝟕)
(C) (𝟗, 𝟎) (D) (𝟗, 𝟕)

Try 8: (1) What is the axis of symmetry of the quadratic function given in the
following table? ___________ (2) Can you find the vertex from the table? ______
(3) Is the maximum value of the function greater than 9? ________

𝒙 −𝟏 𝟎 𝟏 𝟐 𝟑 𝟒

𝒚 −𝟑 𝟓 𝟗 𝟗 𝟓 −𝟑

65
Essential understanding: In the quadratic function 𝒚 = 𝒂𝒙𝟐 + 𝒃𝒙 + 𝒄 (𝒂 ≠ 𝟎) ,
the value of 𝒃 affects the position of the axis of symmetry.

𝟏 𝟏 𝟏
𝒚 = 𝒙𝟐 − 𝟒𝒙 𝒚 = 𝒙𝟐 + 𝟐𝒙 𝒚 = 𝒙𝟐 + 𝟒𝒙
𝟐 𝟐 𝟐

𝒃
Consider the above graphs. Since the axis of symmetry is 𝒙 = − , we get
𝟐𝒂
−𝟒 𝟐 𝟒
𝒙=− 𝟏 =𝟒 𝒙=− 𝟏 = −𝟐 𝒙=− 𝟏 = −𝟒
𝟐• 𝟐• 𝟐•
𝟐 𝟐 𝟐

Notice that the axis of symmetry changes with each change in the 𝒃-value when
the leading coefficient 𝒂 is fixed. When the constant term 𝒄 is missing (𝒄 = 𝟎)
in the quadratic function, the graph goes through the origin (0, 0).

Try 9: Find each axis of symmetry of the following quadratic functions. Please
mark the parabola with its function and draw the axis of symmetry on the graph.
(1) 𝒚 = 𝟐𝒙𝟐 + 𝟒𝒙 − 𝟏

(2) 𝒚 = 𝟐𝒙𝟐 + 𝟐𝒙 − 𝟏

(3) 𝒚 = 𝟐𝒙𝟐 − 𝟓𝒙 − 𝟏

66
Try 10: Below are three graphs of the functions 𝒚 = 𝒂𝒙𝟐 + 𝒃𝒙 + 𝒄 (𝒂 ≠ 𝟎) .
Fill in the blanks with the signs of “ >, = , < ” .
(1) (2) (3)

𝒂 _____ 0 , 𝒂 _____ 0 , 𝒂 _____ 0 ,


𝒃 _____ 0 , 𝒃 _____ 0 , 𝒃 _____ 0 ,
𝒄 _____ 0 , 𝒄 _____ 0 , 𝒄 _____ 0 ,

𝒃𝟐 − 𝟒𝒂𝒄 ____ 0 , 𝒃𝟐 − 𝟒𝒂𝒄 ____ 0 , 𝒃𝟐 − 𝟒𝒂𝒄 ____ 0 .

Try 11: Compare the graph of the function 𝒚 = 𝟐𝒙𝟐 + 𝒃𝒙 + 𝟑 (𝒃 ≠ 𝟎) with


the graph of the function 𝒚 = 𝟐𝒙𝟐 + 𝟑. What is the same and different? _____
(A) They have the same shape and the same positions
(B) They have the same shape, but different positions
(C) They have the different shapes and different positions
(D) They have the different shapes and different vertexes.

Try 12: The graph of the function 𝒚 = 𝒙𝟐 − 𝟐𝒙 + 𝟑 must ______.


(A) have intercepts with x-axis (B) have no intercepts with y-axis
(C) have intercepts with y-axis, but no intercepts with x-axis
(D) have the minimum point of (0, 3)

Try 13: For the graph of the quadratic function 𝒚 = 𝒂𝒙𝟐 + 𝒃𝒙 + 𝒄 (𝒂 ≠ 𝟎) ,


the value of 𝒂 affects __________________________; the value of 𝒃 affects
___________________________; the value of 𝒄 affects _________________.

67
Intersection Points of Graphs

Example 1: Given the functions 𝒇(𝒙) = 𝒙𝟐 and 𝒈(𝒙) = 𝟐𝒙, determine the
values of 𝒙 that will make 𝒇(𝒙) = 𝒈(𝒙) .

Solution: Set up an equation of 𝒇(𝒙) = 𝒈(𝒙), since two functions are equal
at a certain x-value. And then solve the equation for x. That is,
𝒙𝟐 = 𝟐𝒙
𝒙𝟐 − 𝟐𝒙 = 𝟎
𝒙(𝒙 − 𝟐) = 𝟎
𝒙 = 𝟎 or 𝒙 − 𝟐 = 𝟎
𝒙𝟏 = 𝟎, 𝒙𝟐 = 𝟐

Answer: When 𝒙 = 𝟎, 𝟐, the


function 𝒇(𝒙) = 𝒙𝟐 is equal
to the function 𝒈(𝒙) = 𝟐𝒙.

On the graphs, the intersection points of 𝒇(𝒙) and 𝒈(𝒙) are those numbers x
for which 𝒇(𝒙) = 𝒈(𝒙). So we can also find the answer by graphing. From
the above graphs of the two functions, we can see that the two functions have
two intersections points: (0, 0) and (2, 4). So the x-coordinates of the two
intersection points 𝒙 = 𝟎 and 𝒙 = 𝟐 are the answers.

Try 1: Given the functions 𝒉(𝒙) = 𝒙𝟐


and 𝒋(𝒙) = −𝒙 + 𝟐, find the values of
𝒙 for which 𝒉(𝒙) = 𝒋(𝒙). Please find
the answer algebraically and graphically.

68
Example 2: Given 𝒇(𝒙) = 𝟑𝒙𝟐 + 𝟗𝒙 − 𝟖
𝒈(𝒙) = 𝟐𝒙 + 𝟑
solve the equation 𝒇(𝒙) = 𝟐𝒈(𝒙) algebraically for x, to the nearest tenth.

Solution: Since f(𝒙) = 𝟐𝒈(𝒙) , with the substitution method, we get

𝟑𝒙𝟐 + 𝟗𝒙 − 𝟖 = 𝟐(𝟐𝒙 + 𝟑) (Equation model)

𝟑𝒙𝟐 + 𝟗𝒙 − 𝟖 = 𝟒𝒙 + 𝟔
𝟑𝒙𝟐 + 𝟗𝒙 − 𝟖 − (𝟒𝒙 + 𝟔) = 𝟎 (Make the right side be 0.)

𝟑𝒙𝟐 + 𝟓𝒙 − 𝟏𝟒 = 𝟎 (In standard form)

Solve the equation for x using the quadratic formula.


𝒃𝟐 − 𝟒𝒂𝒄 = 𝟓𝟐 − (𝟒) (𝟑)(−𝟏𝟒) = 𝟐𝟓 + 𝟏𝟔𝟖 = 𝟏𝟗𝟑

−𝒃 ± √𝒃𝟐 − 𝟒𝒂𝒄 −𝟓± √𝟏𝟗𝟑 −𝟓 ±𝟏𝟑.𝟖𝟗


𝒙 = = ≈
𝟐𝒂 𝟐×𝟑 𝟔

−𝟓 +𝟏𝟑.𝟖𝟗
Roots: 𝒙𝟏 ≈ ≈ 𝟏. 𝟒𝟖𝟏𝟔𝟕 ≈ 𝟏. 𝟓
𝟔
−𝟓 −𝟏𝟑.𝟖𝟗
𝒙𝟐 ≈ ≈ −𝟑. 𝟏𝟒𝟖𝟑 ≈ −𝟑. 𝟏
𝟔

Try 2: Given the functions 𝒇(𝒙) = 𝒙𝟐 + 𝟓𝒙 − 𝟏 and 𝒈(𝒙) = 𝟑𝒙 + 𝟐 ,


solve the equation 𝒇(𝒙) = 𝟑𝒈(𝒙) algebraically for x, to the nearest tenth.

69
Example 3: When solving the equation 𝒙𝟐 − 𝟐𝒙 − 𝟑 = 𝟎 by graphing, most
students solve it by drawing the graph of the realted function 𝒚 = 𝒙𝟐 − 𝟐𝒙 − 𝟑,
and then find the roots from the x-intercepts on the graph. While Michael sovles
it with a different method. He converts
the equation to 𝒙𝟐 = 𝟐𝒙 + 𝟑. And then
he draws the graphs of two functions
𝒚 = 𝒙𝟐 and 𝒚 = 𝟐𝒙 + 𝟑 shown on the
right. He states that the x-coordinates
of the intersection points of two graphs
are the roots of the original equation,
which are 𝒙 = −𝟏 and 𝒙 = 𝟑.

Think: Does he get the correct answer? Do you think his method is correct or not?
Answer: The method Michael used is correct and his answer is correct, too.
The two functions are equal at the intersection points of the two graphs.

Example 4: A cell phone manufacturer has a monthly production cost at site A


𝟏 𝟏
of 𝑨(𝒙) = 𝒙𝟐 , while the production cost at site B is 𝑩(𝒙) = 𝒙 + 𝟓𝟎, where
𝟒𝟎 𝟐
𝒙 represnts the number of products, in hundred. 𝑨(𝒙) and 𝑩(𝒙) are the production
costs, in hundreds of dollars. If the manufacturer plans on manufacturing 8,000
products per month, which site should they use?

Solution: From the graphs, we can see


the two graphs have two intersection
points about: (−36, 32) and (56, 78).
We ignore the negative parts, since the
products (x) can not be negative. When
0 < x < 56, the cost at site A is lower
than the cost at site B. When x > 56,
the cost at site A is higher than the cost
at site B. 8,000 products is x = 80 on
the graph, so they choose the site B.

70
Practice 4

1. The zero of the function 𝒇(𝒙) = 𝟑𝒙𝟐 − 𝟔𝒙 − 𝟗 are _____________.


(A) 𝟏 and 𝟑 (B) −𝟏 and 𝟑
(C) 𝟏 and −𝟑 (D) −𝟏 and −𝟑

2. If 𝒇(𝒙) = −𝟒𝒙𝟐 + 𝟓𝟐, what is the value of 𝒇(𝟑)? ________


(A) 𝟏𝟔 (B) −𝟏𝟔 (C) 𝟒 (D) ±𝟒
3. Which function has the zero of −𝟏 and 𝟓? ________
(A) (B)

4. Find the zero of the function 𝒇(𝒙) = (𝒙 − 𝟓)𝟐 − 𝟖𝟏, algebraically.

5. The function 𝒉(𝒙) is defined by the expression 𝒙𝟐 + 𝟐𝒙 − 𝟏𝟓.


Use factoring to determine the zeros of 𝒉(𝒙).

71
6. If the graph of the function 𝒚 = 𝒂𝒙𝟐 + 𝒃𝒙 + 𝒄 has one x-intercept (−𝟑, 𝟎).
then the equation 𝒂𝒙𝟐 + 𝒃𝒙 + 𝒄 = 𝟎 has ____ root(s), which is __________.
(A) 1 (B) 2 (C) 𝒙=𝟎 (D) 𝒙 = −𝟑

7. If the equation 𝒂𝒙𝟐 + 𝒃𝒙 + 𝒄 = 𝟎 has two roots 𝒙𝟏 = −𝟔, 𝒙𝟐 = 𝟐, then


the quadratic function 𝒚 = 𝒂𝒙𝟐 + 𝒃𝒙 + 𝒄 has two x-intercepts, which are
_________ and ________. The axis of symmetry of the function is ______.
(A) (𝟎, −𝟔) (B) (𝟎, 𝟐) (C) (−𝟔, 𝟎) (D) (𝟐, 𝟎)
𝟏
(E) y-axis (F) 𝒙 = 𝟐 (G) 𝒙 = −𝟐 (H) 𝒙 =
𝟐

8. Answer the following questions without graphing the functions.


(1) The graph of the function 𝒚 = 𝒙𝟐 − 𝟑𝒙 + 𝟒 has ______ x-intercept(s).
The coordinates of the x-intercept(s) are ________________________.

(2) The graph of the function 𝒚 = 𝒙𝟐 + 𝟐𝒙 − 𝟖 has ______ x-intercept(s).


The coordinates of the x-intercept(s) are ________________________.

9. If the graph of the function 𝒚 = 𝒂𝒙𝟐 passes through the point of (−𝟐, 𝟔),
then the graph also passes through the point of ________.
(A) (−𝟐, −𝟔) (B) (𝟐, 𝟔) (C) (−𝟔, 𝟐) (D) (𝟔, −𝟐)
𝟏
10. If 𝒇(𝒙) = − 𝒙𝟐 + 𝟓, what is the function of 𝒈(𝒙) = 𝟒𝒇(𝒙)? ________
𝟒

(A) 𝒈(𝒙) = −𝟒𝒙𝟐 + 𝟓 (B) 𝒈(𝒙) = −𝟒𝒙𝟐 + 𝟐𝟎


(C) 𝒈(𝒙) = −𝒙𝟐 + 𝟓 (D) 𝒈(𝒙) = −𝒙𝟐 + 𝟐𝟎

11. Below is the table of values of 𝒚 = 𝒂𝒙𝟐 + 𝒃𝒙 + 𝒄 . The data pairs are close
to the x-intercept. The root of 𝒂𝒙𝟐 + 𝒃𝒙 + 𝒄 = 𝟎 is 𝒙 = ________.
𝒙 −𝟒. 𝟓 −𝟒. 𝟒 −𝟒. 𝟑 −𝟒. 𝟐 −𝟒. 𝟏

𝒚 𝟎. 𝟕𝟓 𝟎. 𝟑𝟔 −𝟎. 𝟎𝟏 −𝟎. 𝟑𝟔 −𝟎. 𝟔𝟗

72
𝟏 𝟏
12. If 𝒇(𝒙) = 𝒙𝟐 − ( 𝒙 + 𝟓), what is the value of 𝒇(𝟒)? ________
𝟖 𝟐
(A) 𝟑. 𝟓 (B) 𝟓 (C) −𝟑. 𝟓 (D) −𝟓

13. There is the function 𝒇(𝒙) = 𝟑𝒙𝟐 + 𝟒𝒙 − 𝟐 .


(1) State two different methods that could be used to solve the equation
𝒇(𝒙) = 𝟎.

(2) Using one of methods stated in part (1), solve the equation 𝒇(𝒙) = 𝟎
for 𝒙, to the nearest tenth.

𝟏
14. Given the functions 𝒇(𝒙) = 𝒙𝟐 + 𝟑𝒙 − 𝟏𝟎 and 𝒈(𝒙) = 𝒙 − 𝟑, find the
𝟐
values of 𝒙 for which 𝒇(𝒙) = 𝒈(𝒙)? _____ Please write down the process.
(A) 𝟏. 𝟒𝟐𝟐 and −𝟒. 𝟗𝟐𝟐 (B) 𝟏. 𝟔𝟕𝟔 and −𝟒. 𝟏𝟕𝟔
(C) −𝟏. 𝟒𝟐𝟐 and −𝟒. 𝟗𝟐𝟐 (D) −𝟏. 𝟔𝟕𝟔 and −𝟒. 𝟏𝟕𝟔

73
15. If 𝒖(𝒙) = 𝒙𝟐 and 𝒗(𝒙) = −𝒙, determine all value(s) of 𝒙 that satisfy
𝒖(𝒙) = 𝟐𝒗(𝒙) algebraically and graphically.

74
16. A manufacturer of laptop computers has a monthly production cost at site A
𝟏
of 𝑨(𝒙) = 𝒙𝟐 , while the production cost at site B is 𝑩(𝒙) = 𝟐𝒙 + 𝟏𝟓, where
𝟏𝟎
𝒙 represnts the number of products, in hundreds, 𝑨(𝒙) and 𝑩(𝒙) are the
production costs, in hundreds of dollars.

(1) Graph the production cost functions on the coordinate plane below and label
them site A and site B.
(2) State the positive value(s) of x
for which the production costs at
the two sites are equal. Explain
how you determined your answer.
(3) If the manufacturer plans on
manufacturing 800 products
per month, which site should
they use?

17. On the right is the graph of the


quadratic function 𝒚 = 𝒂𝒙𝟐 + 𝒃𝒙 + 𝒄.

(1) The related equation has two roots,


which are 𝒙 = _____ , 𝒙 = _____.
(2) When 𝒙 = _________, 𝒚 = 𝟎.
(3) When 𝒙 < −𝟏 or 𝒙 > 𝟑, 𝒚 ____ 𝟎 .
(4) When −𝟏 < 𝒙 < 𝟑, 𝒚 ____ 𝟎 .

75
18. Samuel and Kelly are each saving money for vacation. The total amount of
money Samuel will save is given by the function𝒇(𝒙) = 𝟕𝒙 + 𝟓𝟎. The total
amount of money Kelly will save is given by the function 𝒈(𝒙) = 𝒙𝟐 + 𝟑𝟐.
After how many weeks, x, will they have the same amount of money saved?
Explain how you arrived at your answer.

19. Solve the equation −𝒙𝟐 − 𝟓𝒙 + 𝟐 = 𝟎 by graphing.


If integer roots can not be found, estimate the roots to the nearest tenth.

76
Real Word Problems
Involving Quadratic Functions

Example 1: A company earns a weekly profit of 𝑷 dollars by selling 𝒙 items,


according to the function 𝑷(𝒙) = −𝟎. 𝟓𝒙𝟐 + 𝟔𝟎𝒙 − 𝟏𝟎𝟎. How many items
does the company have to sell each week to maximize the profit?

Solution: Since 𝒂 < 𝟎, the function has a maximum, which must occur at
𝒃
the vertex. Use − to find x-coordinate of the vertex, which is the answer.
𝟐𝒂
𝒃 𝟔𝟎
𝒙=− =− = 𝟔𝟎 items
𝟐𝒂 𝟐 •(− 𝟎.𝟓)

Answer: The company has to sell 60 items each week to maximize the profit.

Try 1: A manufacturer of electric space heaters has daily production costs of


𝑪(𝒙) = 𝟕𝟐𝟎𝟎 − 𝟒𝟎𝒙 + 𝟎. 𝟐𝟓𝒙𝟐 , where 𝑪 is the total cost (in dollars) and 𝒙
is the number of units produced. How many units should be produced each
day to yield a minimum cost? What is the minimum cost?

77
Example 2: A football player kicks a football from the ground. The path of the
𝟏 𝟒
football can be modeled by the function 𝒉(𝒙) = − 𝒙𝟐 + 𝒙 , where 𝒙 is the
𝟏𝟐𝟓 𝟓
horizontal distance from the kick; 𝒉(𝒙) is the height of the football above the
ground, when both measured in feet.
(1) What is the maximum height the football can reach?
(2) Determine the vertex of 𝒚 = 𝒉(𝒙). (3) How far is the football kicked out
when it hits the ground?

h(x)

Solution: Since the path of the football is a parabola, we can set up a coordinate
plane for it. Let the beginning kick point be the origin of the coordinate plane.
𝟏 𝟒 𝟏 𝟒
The function is given: 𝒉(𝒙) = − 𝒙𝟐 + 𝒙 , where 𝒂 = − , 𝒃 = , 𝒄 = 𝟎.
𝟏𝟐𝟓 𝟓 𝟏𝟐𝟓 𝟓

The maximum height the football can reach occurs at the vertex.
The value of the axis of symmetry is the x-coordinate of the vertex.
𝟒
𝒃
(1) The axis of symmetry: 𝒙 = − =− 𝟓
𝟏 = 𝟓𝟎 ft.
𝟐𝒂 𝟐 •(− )
𝟏𝟐𝟓

(2) The maximum or minimum value is the y-coordinate of the vertex.


𝟏 𝟒
The maximum height is: 𝒚 = 𝒉(𝟓𝟎) = (− )(𝟓𝟎)𝟐 + ( )(𝟓𝟎) = 𝟐𝟎 ft.
𝟏𝟐𝟓 𝟓
𝟏 𝟒
We can also find the maximum 𝟒𝒂𝒄−𝒃𝟐 𝟒 •(− )• 𝟎− ( )𝟐
𝟏𝟐𝟓 𝟓
= 𝟏 = 𝟐𝟎 ft.
value by the formula: 𝟒𝒂 𝟒 •(− )
𝟏𝟐𝟓

Answer: The vertex is (𝟓𝟎, 𝟐𝟎). The maximum height is 20 feet.

78
𝟏 𝟒
(3) When the football hits the ground, the y-value is 0. So 𝟎 = − 𝒙𝟐 + 𝒙
𝟏𝟐𝟓 𝟓
𝟏 𝟒
Solve the equation for 𝒙 by factoring 𝒙(− 𝒙+ )=𝟎
𝟏𝟐𝟓 𝟓
𝟏 𝟒
Roots: 𝒙𝟏 = 𝟎 , or − 𝒙 + = 𝟎 → 𝒙𝟐 = 𝟏𝟎𝟎
𝟏𝟐𝟓 𝟓

Answer: The first root is 0 feet, which is the starting point when the football was
kicked out. So we reject the root of 𝒙𝟏 = 𝟎 and choose the second root. Then the
football is 100 feet far away from the kick point when it hits the ground.

Example 3: In example 2, the football falls down after the maximum height.
Can you graph the path of the football falling down from the highest point?

Solution: From the maximum height, the football goes through the interval of
the horizontal distance between the vertex and the x-intercept, 𝟓𝟎 ≤ 𝒙 ≤ 𝟏𝟎𝟎.
We can find four points to graph it. Since we already got the vertex and the
ending point, let’s choose two more points between them 𝒙 = 𝟕𝟎, 𝒙 = 𝟖𝟓.
𝟏 𝟒
The function is given: 𝒚 = 𝒉(𝒙) = − 𝒙𝟐 + 𝒙
𝟏𝟐𝟓 𝟓

𝟏 𝟒
(1) When 𝒙 = 𝟕𝟎 , 𝒚=− (𝟕𝟎)𝟐 + (𝟕𝟎) = 𝟏𝟔. 𝟖 ft
𝟏𝟐𝟓 𝟓
𝟏 𝟒
(2) When 𝒙 = 𝟖𝟓 , 𝒚=− (𝟖𝟓)𝟐 + (𝟖𝟓) = 𝟏𝟎. 𝟐 ft
𝟏𝟐𝟓 𝟓

So the two points are (𝟕𝟎, 𝟏𝟔. 𝟖), (𝟖𝟓, 𝟏𝟎. 𝟐) . The other two points are
the vertex of (𝟓𝟎, 𝟐𝟎) and the x-intercept (𝟏𝟎𝟎, 𝟎). We get the graph below.

𝒉(𝒙)

(𝟎, 𝟎) x

79
Try 2: A golf course uses sprinkling system
to irrigate the lawn. Water is sprayed out
from the sprinkler on the ground. The path
of the sprayed water can be modeled by the
𝟏 𝟑
function 𝒉(𝒙) = − 𝒙𝟐 + 𝒙 , where 𝒙 is
𝟐𝟓 𝟓
the horizontal distance from the sprinkler;
𝒉(𝒙) is the height of the water above the ground, when both measured in feet.

(1) The designer wants to know how far the water can reach.
(2) What is the maximum height the water can reach?
(3) Determine the vertex of 𝒚 = 𝒉(𝒙).
(4) The path of the sprayed water looks like a parabola. However, the path is not
clear to see after the maximum height it can reach. Can you graph the path from
the maximum height to the end on the coordinate?

80
Example 4: A ball is shot into the air from a platform, which is 80 feet above the
ground. The ball is launched at an initial velocity of 64 feet per second. The
function 𝒉 = −𝟏𝟔𝒕𝟐 + 𝟔𝟒𝒕 + 𝟖𝟎 represents the height of the ball after 𝒕 seconds.
(1) Find the time it takes the ball to reach the maximum height?
(2) Determine the realistic domain for the function.

Solution: The maximum occur at the vertex. Let’s find the vertex.
𝒃 𝟔𝟒
(1) 𝒕=− =− = 𝟐 seconds
𝟐𝒂 𝟐 •(− 𝟏𝟔)
𝟒𝒂𝒄−𝒃𝟐 𝟒 •(− 𝟏𝟔)• 𝟖𝟎 − (𝟔𝟒 )𝟐
When 𝒕 = 𝟐 , the maximum is: = = 𝟏𝟒𝟒 ft.
𝟒𝒂 𝟒 •(− 𝟏𝟔)

Answer: At 𝟐 seconds, the ball reaches its maximum height of 144 feet.

(2) The domain of a function is the set of


all values of 𝒙. The domain in this context
is from the starting point to the ending point.
The starting point is at 𝒕 = 𝟎 seconds.
At the ending point, the ball hits the ground,
where 𝒉 = 𝟎. So we need find the time 𝒕
when 𝒉 = 𝟎.

Let’s solve the equation 𝟎 = −𝟏𝟔𝒕𝟐 + 𝟔𝟒𝒕 + 𝟖𝟎 to find the time when the ball
hits the ground. With the quadratic formula, we get two roots: 𝒕 = 𝟓, 𝒕 = −𝟏.
Negative time won’t work in this word problem. So we choose 𝒕 = 𝟓. That is,
At 𝟓 seconds, the ball hits the ground. Answer: The domain of the function is
𝟎 ≤ 𝒕 ≤ 𝟓 seconds. How do we get the above function?

𝒉 = −𝟏𝟔𝒕𝟐 + 𝟔𝟒𝒕 + 𝟖𝟎

Gravity pulls it down, The initial height is 80


The ball is launched at
by about 16 feet per feet above the ground.
an initial velocity of
second squared
64 feet per second

81
Try 3: A ball is shot into the air from the top of a building, which is 64 feet
above the ground. The ball is launched at a velocity of 48 feet per second. The
function 𝒉 = −𝟏𝟔𝒕𝟐 + 𝟒𝟖𝒕 + 𝟔𝟒 represents the height of the ball after 𝒕
seconds. (1) Find the time it takes the ball to reach the maximum height?
(2) Determine the realistic domain for the function.

Try 4: There is a free fall experiment. An object is dropped from the edge of
𝟏
a cliff, that is 𝟐𝟔𝟎 feet above the ground. The Free Fall Formula is 𝒉 = 𝒈𝒕𝟐 ,
𝟐
where 𝒉 represents the height and 𝒕 represents the time after it is dropped.
𝟏
𝒈 represents the gravity which is a constant of 16 feet per second squared.
𝟐
Since the object is launched at an initial velocity of 0 feet per second, the
function 𝒉(𝒕) = −𝟏𝟔𝒕𝟐 + 𝟐𝟔𝟎 can be used to model the free fall experiment.
(1) How long it takes the object to hit the ground? Round to the nearest second.
(2) Determine the approximate domain for the function.

82
Try 5: A model rocket is launched from the ground straight upward.
The height of the rocket above the ground, in feet, is given by the function
𝒉(𝒕) = −𝟏𝟔𝒕𝟐 + 𝟏𝟔𝟎𝒕 , where 𝒕 represents the time in seconds, since the
rocket was launched.
(1) Can the rocket reach the height of 300 feet?
(2) Can the rocket reach the height of 400 feet?
(3) Can the rocket reach the height of 500 feet?
(4) State the time interval, in seconds, during which the height of the rocket
decreases. ________ (Choice question.) Explain your reasoning.
(A) 𝟎 ≤ 𝒕 ≤ 𝟓 (B) 𝟓 ≤ 𝒕 ≤ 𝟏𝟎 (C) 𝟎 ≤ 𝒕 ≤ 𝟏𝟎
(5) There is no constant term in the function. Why?

83
Example 5: Kevin wants to make a rectangular garden with an 80-feet fence.
The area of the rectangular garden changes as the length of the fenced rectangle
changes. How long is the length of the rectangle, when the area of the garden is
a maximum?

Hint: Find the function that model the relationship of the area and the length of
the fenced rectangle first. Then find the maximum value of the function.
Solution: (1) Let the length of the fenced rectangle be 𝒍 .
Let the width of the fenced rectangle be 𝒘.
The length of the fence is 80 feet, which is the circumference of the rectangle.
Then the width of the fenced rectangle is 𝟖𝟎 = 𝟐𝒍 + 𝟐𝒘 𝒘 = 𝟒𝟎 − 𝒍 .

(2) The function model is : 𝑺 = 𝒍(𝟒𝟎 − 𝒍) 𝑺 = −𝒍𝟐 + 𝟒𝟎𝒍


𝒃
(3) When 𝒍 = − , the fenced rectangular garden has the maximum area:
𝟐𝒂
𝒃 𝟒𝟎
𝒍=− =− = 𝟐𝟎 feet
𝟐𝒂 𝟐 •(− 𝟏)

(4) The maximum area: 𝑺 = −𝒍𝟐 + 𝟒𝟎𝒍 = −(𝟐𝟎)𝟐 + 𝟒𝟎(𝟐𝟎) = 𝟒𝟎𝟎 sq ft.

Answer: When the length of the fenced garden is 20 feet, the fenced garden
has a maximum area of 400 feet.

Try 6: Wendy wants to make a rectangular garden with an 60-feet fence.


Can she make a rectangular garden with the area of 250 square feet?

84
Try 7: Tim wants to fence in a rectangular
Wall of the garage
portion of his back yard against the back of
his garage for a rectangular vegetable garden. Garden W
He plans to use a 60-feet fence and needs
fence only three sides. How long is the width
of the rectangular garden when the area of the 𝟔𝟎 − 𝟐𝑾
garden is a maximum? Find the maximum area of the garden.
Hint: The lengths of the 3 fenced sides of the rectangle must add up to 60.

Try 8: Jennifer wants to make a rectangular garden with a 100-feet fence.


(1) Can she make a rectangular garden that has the area of 400 square feet?
(2) Can she make a rectangular garden that has the area of 700 square feet?
(3) What is the maximum area of the rectangular garden she can make?
(4) If she wants to make the rectangular garden that has the area of 400 square
feet, what is the length and width of the rectangular garden?

85
Practice 5

1. A company’s weekly revenue in dollars is given by the function


𝑹(𝒙) = 𝟔, 𝟎𝟎𝟎𝒙 − 𝟑𝒙𝟐 , where 𝒙 is the number of items produced during
a week. What amount of items will produce the maximum revenue?

2. A manufacturer of LED light bulbs has a daily cost of the function


𝑪(𝒙) = 𝟑, 𝟎𝟎𝟎 − 𝟏𝟎𝒙 + 𝟎. 𝟎𝟏𝒙𝟐 , where 𝑪 is the total cost (in dollars)
and 𝒙 is the number of light bulbs produced. What number of light bulbs
will be produced in the minimum cost? What is the minimum cost?

3. The function 𝒉 = −𝟓𝒕𝟐 + 𝟏𝟗𝒕 + 𝟒 represents the height, in meters, of a


ball 𝒕 seconds after launch. The initial height of the ball, in meters, is ______.
(A) −𝟓 (B) 𝟏𝟗 (C) 𝟒 (D) 𝟎

4. A ball is thrown into the air from the top of a building, which is 95 feet
above the ground. The ball is launched at 48 feet per second. The function
𝒉(𝒕) represents the height of the ball after 𝒕 seconds. Please fill in the
blanks to complete the function: 𝒉(𝒕) = −𝟏𝟔𝒕𝟐 + ______ + _____

5. Advertising revenue for newspapers in the United States for the years 1985
through 1999 is approximated by the model 𝑹 = −𝟏. 𝟎𝟑 + 𝟕. 𝟏𝟏𝒕 − 𝟎. 𝟑𝟖𝒕𝟐 ,
where R is revenue in billions of dollars and t represents the year with 𝒕 = 𝟓
corresponding to the year 1985. In what year will revenue be maximum?

86
6. Benjamin launched a ball into the air. The height of the ball can be represented
by the function 𝒉 = −𝟓𝒕𝟐 + 𝟏𝟑𝒕 + 𝟔, where 𝒉 is the height, in units, and 𝒕 is
the time, in seconds, after the ball was launched.
(1) Graph the function from 𝒕 = 𝟎 to 𝒕 = 𝟒 seconds. (2) State the coordinates
of the vertex and explain its meaning in the context of the problem.

87
Modeling Quadratic Functions

Try 1: Write 𝒇(𝒙) = (𝒙 − 𝟏) (𝒙 − 𝟓)


in standard form. What are the roots of
the related quadratic equation?

Given two x-intercepts of a quadratic function, can you find the quadratic
function in standard form? Hint: We have learned how to factor the quadratic
equation 𝒂𝒙𝟐 + 𝒃𝒙 + 𝒄 = 𝟎 with two given roots of 𝒙𝟏 and 𝒙𝟐 , and we
get the formula:
𝒂𝒙𝟐 + 𝒃𝒙 + 𝒄 = 𝒂(𝒙 − 𝒙𝟏 ) (𝒙 − 𝒙𝟐 )

Example 1: Below is the graph of a quadratic function. It has two x-intercepts


(−𝟏, 𝟎) and (𝟑, 𝟎). Find the equation of the quadratic function from the graph.

Solution: From the x-intercepts,


we know that the related quadratic
equation has two roots 𝒙𝟏 = −𝟏
and 𝒙𝟐 = 𝟑. According to the
above formula, we get step 1:
𝒚 = 𝒂(𝒙 − 𝒙𝟏 ) (𝒙 − 𝒙𝟐 )
= 𝒂(𝒙 − (−𝟏))(𝒙 − 𝟑)
= 𝒂(𝒙𝟐 − 𝟐𝒙 − 𝟑)

If we know one more point on the parabola, then we can find the coefficient 𝒂 .

88
Step 2: Given one more point of (𝟐, −𝟔), that is 𝒙 = 𝟐 and 𝒚 = −𝟔,
we can substitute them into the function we got in step 1.
𝒚 = 𝒂(𝒙𝟐 − 𝟐𝒙 − 𝟑)
−𝟔 = 𝒂(𝟐𝟐 − 𝟐 • 𝟐 − 𝟑)
−𝟔 = 𝒂(−𝟑)
𝒂=𝟐

Step 3: Substitute 𝒂 = 𝟐 into the function we got in step 1. Then we get


𝒚 = 𝒂(𝒙𝟐 − 𝟐𝒙 − 𝟑)
= 𝟐(𝒙𝟐 − 𝟐𝒙 − 𝟑)
= 𝟐𝒙𝟐 − 𝟒𝒙 − 𝟔

Answer: The quadratic function is 𝒚 = 𝟐𝒙𝟐 − 𝟒𝒙 − 𝟔.

Example 2: Below is the graph of a quadratic function. It has the vertex of


(𝟏, −𝟖). Given one more point of (𝟐, −𝟔), determine the equation of the
quadratic function from the graph.

Solution: Substitute the vertex into


the quadratic function in vertex form.
Step 1: 𝒚 = 𝒂(𝒙 − 𝒉)𝟐 + 𝒌
𝒚 = 𝒂(𝒙 − 𝟏)𝟐 + (−𝟖)

Step 2: To find 𝒂, substitute the other


point (𝟐, −𝟔) into the above function.

𝒚 = 𝒂(𝒙 − 𝟏)𝟐 − 𝟖
𝒚 = 𝒂(𝒙𝟐 − 𝟐𝒙 + 𝟏) − 𝟖
−𝟔 = 𝒂(𝟐𝟐 − 𝟐 • 𝟐 + 𝟏) − 𝟖
𝒂=𝟐
Step 3: Substitute 𝒂 = 𝟐 into the above function 𝒚 = 𝒂(𝒙 − 𝟏)𝟐 − 𝟖. We get
𝒚 = 𝟐(𝒙 − 𝟏)𝟐 + (−𝟖) and finally get 𝒚 = 𝟐𝒙𝟐 − 𝟒𝒙 − 𝟔
89
Example 3: Below is the graph of a quadratic function. It has the y-intercept
of (𝟎, −𝟔). Given two more points of (𝟒, 𝟏𝟎) and (𝟑. 𝟓, 𝟒. 𝟓), determine
the equation of the quadratic function from the graph.

Solution: (1) From the y-intercept,


we know that 𝒄 = −𝟔. Then we get
the function 𝒚 = 𝒂𝒙𝟐 + 𝒃𝒙 − 𝟔

(2) Substitute the point of (𝟒, 𝟏𝟎)


into the above function. We get
𝟏𝟎 = 𝒂 • 𝟒𝟐 + 𝟒𝒃 − 𝟔
𝟏𝟔𝒂 + 𝟒𝒃 = 𝟏𝟔
𝟒𝒂 + 𝒃 = 𝟒

Substitute the point of (𝟑. 𝟓, 𝟒. 𝟓) into the same function and then we get
𝟒. 𝟓 = 𝒂 • 𝟑. 𝟓𝟐 + 𝟑. 𝟓𝒃 − 𝟔
𝟏𝟐. 𝟐𝟓𝒂 + 𝟑. 𝟓𝒃 = 𝟏𝟎. 𝟓
𝟑. 𝟓𝒂 + 𝒃 = 𝟑

(3) Form a system of equations. And then solve it for 𝒂 and 𝒃.


𝟒𝒂 + 𝒃 = 𝟒 ①
𝟑. 𝟓𝒂 + 𝒃 = 𝟑 ②

Let’s solve the system of equations by ① − ②. We get 𝟎. 𝟓𝒂 = 𝟏 → 𝒂 = 𝟐


Substitute 𝒂 = 𝟐 into the equation ①, and then we get
𝟒•𝟐+𝒃=𝟒 → 𝒃 = −𝟒

(4) So far, we get 𝒂 = 𝟐, 𝒃 = −𝟒 and 𝒄 = −𝟔.


Substitute them into the quadratic function 𝒚 = 𝒂𝒙𝟐 + 𝒃𝒙 + 𝒄
and then we get 𝒚 = 𝟐𝒙𝟐 − 𝟒𝒙 − 𝟔

Answer: The quadratic function is 𝒚 = 𝟐𝒙𝟐 − 𝟒𝒙 − 𝟔.

90
Practice 6

1. There are three situations to model quadratic functions.


(1) When we use 𝒚 = 𝒂(𝒙 − 𝒉)𝟐 + 𝒌 to find the quadratic functions, we need
know 2 points on the parabola, in which we must know the point of _____.
(2) When we use 𝒚 = 𝒂(𝒙 − 𝒙𝟏 ) (𝒙 − 𝒙𝟐 ) to find the quadratic functions, we
need know 3 points of the graph, in which we must know the points of ___.
(3) When we use 𝒚 = 𝒂𝒙𝟐 + 𝒃𝒙 + 𝒄 to find the quadratic functions, we need
know ___ (1, 2, 3) points on the parabola.
(A) the vertex (B) the x-intercepts (C) the y-intercept

2. Given two points on the parabola of a certain quadratic function: (−𝟏, 𝟏𝟑)
and the vertex (𝟐, −𝟓), can you find the equation of the quadratic function
from the graph.

Step 1: Since there is a vertex (𝟐, −𝟓), we get 𝒚 = 𝒂(𝒙 − ____)𝟐 + ____

Step 2: Substitute the point of (−𝟏, 𝟏𝟑) into the above function,
we get ____________________. And get 𝒂 =______

Step 3: Finally, we get 𝒚 = 𝒂(𝒙 − 𝒉)𝟐 + 𝒌 .


𝒚 =_______________
𝒚 =_______________

3. Given a quadratic function below, what is the quadratic function in standard


form? 𝒙 −𝟏 𝟐 𝟑 𝟒 𝟕

𝒚 𝟏𝟐 −𝟑 −𝟒 −𝟑 𝟏𝟐

91
4. There is a quadratic function with two x-intercepts: (−𝟒, 𝟎) and (𝟐, 𝟎).
And its parabola goes through the point of (−𝟔, 𝟖). Can you find the
quadratic function of the parabola?
Step 1: Since there are two x-intercepts, we get 𝒚 = 𝒂(𝒙 − ___) (𝒙 − ___)

Step 2: Substitute the point of (−𝟔, 𝟖) into the above function,


we get ____________________. And get 𝒂 =______
Step 3: Finally, we get 𝒚 =_______________
𝒚 =_______________
𝒚 =_______________

5. Given a quadratic function has the y-intercept: (𝟎, 𝟑). And its parabola goes
through the points of (−𝟐, −𝟓) and (1, 𝟒). Can you find the equation of the
quadratic function from the graph.
Step 1: Since there is a y-intercept: (𝟎, 𝟑), we get 𝒚 = 𝒂𝒙𝟐 + 𝒃𝒙 + _____
Step 2: Substitute the point of (𝟏, 𝟒) into the above function. We get

Substitute the point of (−𝟐, −𝟓) into the above function. We get

Step 3: Form a system of equations with what we got. And solve it for 𝒂 and 𝒃.
_______________
_______________

Step 4: We get 𝒂 =_____, 𝒃 =_____, 𝒄 =_____.


Substitute them into the quadratic function 𝒚 = 𝒂𝒙𝟐 + 𝒃𝒙 + 𝒄
and then we get the final quadratic function 𝒚 =_______________

92
6. Given the parabola of a quadratic function goes through the points of (𝟏, 𝟎),
(𝟕, 𝟎) and (𝟐, 𝟓), please find the equation of the quadratic function from
the graph.

7. Given the parabola of a quadratic function goes through the points of (𝟎, 𝟐),
(𝟏, 𝟏) and (𝟑, 𝟓), please find the equation of the quadratic function from
the graph.

93
8. Given the parabola of a quadratic function goes through the points of (𝟎, −𝟏),
(𝟏, 𝟎) and (−𝟏, 𝟐), please find the equation of the quadratic function from
the graph.

9. Given the table of values of a quadratic function below,


please find the equation of the quadratic function from the table.
𝒙 −𝟐 −𝟏 𝟎 𝟏 𝟐

𝒚 −𝟒 𝟎 𝟐 𝟐 𝟎

94
10. On the right is the graph of
a quadratic function. Please find
the equation of the quadratic
function from the graph.

11. If the quadratic equation 𝒙𝟐 + 𝒃𝒙 + 𝒄 = 𝟎 has two roots 𝒙𝟏 = 𝟑,


𝒙𝟐 = 𝟓, then the related quadratic function is _________.
(A) 𝒚 = (𝒙 + 𝟑) (𝒙 + 𝟓) (B) 𝒚 = (𝒙 − 𝟑) (𝒙 − 𝟓)
(C) 𝒚 = (𝒙 + 𝟑) (𝒙 − 𝟓) (D) 𝒚 = (𝒙 − 𝟑) (𝒙 + 𝟓)

12. If a quadratic equation has two roots 𝒙𝟏 = 𝟑, 𝒙𝟐 = −𝟏 and the leading


coefficient of the related quadratic function is 𝟐, then the function is _____.
(A) 𝒚 = 𝟐(𝒙 + 𝟑) (𝒙 − 𝟏) (B) 𝒚 = 𝟐(𝒙 − 𝟑) (𝒙 + 𝟏)
(C) 𝒚 = 𝟔(𝒙 + 𝟑) (𝒙 − 𝟏) (D) 𝒚 = 𝟔(𝒙 − 𝟑) (𝒙 + 𝟏)

13. The graph of the quadratic function 𝒚 = 𝒙𝟐 + 𝒃𝒙 + 𝒄 (𝒂 ≠ 𝟎) has two


x-intercepts: (𝟐 , 𝟎 ) and (𝟕 , 𝟎). The related equation could be _____ .
(A) 𝒚 = (𝒙 + 𝟐)(𝒙 + 𝟕) (B) 𝒚 = (𝒙 − 𝟐)(𝒙 − 𝟕)
(C) 𝒚 = (𝒙 + 𝟐)(𝒙 − 𝟕) (D) 𝒚 = (𝒙 − 𝟐)(𝒙 + 𝟕)

95
Linear, Quadratic, and
Exponential Models

So far, we have learned 3 kinds of functions: Linear, quadratic and exponential


functions. We can use them to model some sets of data.

Linear Function Quadratic Function Exponential

Example 1: Graph each of the following set of points to see which model is
most appropriate for each set.
(1) (−𝟏, 𝟓), (𝟎, 𝟎), (𝟐, −𝟒), (𝟒, 𝟎), (𝟓, 𝟓).
(2) (−𝟓, −𝟑. 𝟗𝟗), (𝟐, −𝟑), (𝟒, 𝟎), (𝟓, 𝟒), (𝟔, 𝟏𝟐).
(3) (−𝟓, −𝟕), (−𝟑, −𝟑), (𝟎, 𝟑), (𝟐, 𝟕).

(1) Quadratic Model (2) Exponential Model (3) Linear Model

96
Try 1: Graph each of the following set of points to see which model is
most appropriate for each set.
(1) (−𝟐, 𝟑), (𝟎, −𝟑), (𝟐, −𝟓), (𝟒, −𝟑), (𝟔, 𝟑).
(2) (−𝟔, −𝟑), (𝟏, −𝟐), (𝟑, 𝟏), (𝟒, 𝟓), (𝟔, 𝟏𝟐).
(3) (−𝟖, 𝟖), (−𝟒, 𝟔), (𝟎, 𝟒), (𝟒, 𝟐) , (𝟖, 𝟎).

(1) ________________ Model (2) _________________ Model

(3) ________________ Model

97
Essential understanding: When the x-values in a set of data pairs have a common
difference, you can analyze data numerically to find the best model. You can use a
linear function to model data pairs with y-values that have a common difference.
You can use an exponential function to model data pairs with y-values that have a
common ratio.

𝒚 = 𝟒𝒙 + 𝟑 𝒚 = 𝟐𝒙

𝒙 𝒚 𝒙 𝒚
−𝟐 −𝟓 𝟎 𝟏
+𝟏 +𝟒 +𝟏 ×𝟐
−𝟏 −𝟏 𝟏 𝟐
+𝟏 +𝟒 +𝟏 ×𝟐
𝟎 𝟑 𝟐 𝟒
+𝟏 +𝟒 +𝟏 ×𝟐
𝟏 𝟕 𝟑 𝟖
+𝟏 +𝟒 +𝟏 ×𝟐
𝟐 𝟏𝟏 𝟒 𝟏𝟔

A linear model fits the data, A exponential model fits the data,
because the y-values of the table because the y-values of the table
have a common difference of 4. have a common ratio of 2.

𝒙 𝒚 First difference

−𝟐 𝟓 𝒚 = 𝒙𝟐 − 𝟐𝒙 − 𝟑
+𝟏 +𝟓
+𝟐
−𝟏 𝟎 For quadratic functions,
+𝟏 +𝟑
𝟎 −𝟑 +𝟐 the second difference
+𝟏 +𝟏 are constant.
𝟏 −𝟒 +𝟐
+𝟏 −𝟏
𝟐 −𝟑
Second difference

A quadratic model fits the above data, because the second difference of the
y-values are all 2.

98
Example 2: Choose a model using successive differences or ratios. Look at the
table of values to determine which kind of model best describes the data.
𝟐 𝟐 𝟐 𝟐

(1) 𝒙 −𝟒 −𝟐 𝟎 𝟐 𝟒

𝒚 −𝟏𝟎 −𝟒 𝟐 𝟖 𝟏𝟐

First differences: 𝟔 𝟔 𝟔 𝟔

Since the first differences of y-values are constant, the table of values represents
a linear function. Since the first differences is 3, we get the slope of the linear
function 𝒎 = 𝟑.
𝟐 𝟐 𝟐 𝟐

(2) 𝒙 −𝟐 𝟎 𝟐 𝟒 𝟔

𝒚 𝟔 −𝟒 −𝟔 𝟎 𝟏𝟒

First differences: −𝟏𝟎 −𝟐 𝟔 𝟏𝟒

Second differences: 𝟖 𝟖 𝟖

Since the second differences of y-values are all the same, the table of values
represents a quadratic function. Then we can model the data by the methods
we learned in the last section.

𝟏 𝟏 𝟏 𝟏

(3) 𝒙 −𝟏 𝟎 𝟏 𝟐 𝟑
𝟏
𝒚 −𝟑 𝟏 𝟑 𝟗 𝟐𝟕

𝟐
First differences: 𝟐 𝟔 𝟏𝟖 (Not constant)
𝟑

Ratios: 𝟑 𝟑 𝟑 𝟑 (Ratios are constant)

Since the ratios of y-values are all the same, the table of values represents
an exponential function. The constant ratio is 3, which is the value of the base.
𝒃 = 𝟑.

99
Try 2: Look for a pattern in each table to determine which type of function best
models the data? Use successive differences or ratios.

(1) 𝒙 𝟎 𝟏 𝟐 𝟑 𝟒

𝒚 −𝟓 −𝟑 −𝟏 𝟏 𝟑

(2) 𝒙 𝟎 𝟏 𝟐 𝟑 𝟒

𝒚 𝟔 𝟐 𝟎 𝟎 𝟐

Try 3: Which type of function best models the data in each table?

(1) 𝒙 𝒚 (2) 𝒙 𝒚
−𝟑 𝟏𝟖 −𝟏 𝟖
−𝟐 𝟔 𝟎 𝟒
−𝟏 𝟐 𝟏 𝟐
𝟎 𝟔 𝟐 𝟏
𝟏 𝟏𝟖 𝟑 𝟎. 𝟓

Try 4: Determine which kind of functions model the following data best first.
Then write an equation for the function that models the data.

(1) 𝒙 𝟏 𝟐 𝟑 𝟒 𝟓

𝒚 𝟎 −𝟑 −𝟒 −𝟑 𝟎

100
Example 3: The number of visitors at a Website over several days is shown
in the following table. Determine which model fits the data best. Then write
a function that models the data.

Days 𝟏 𝟐 𝟑 𝟒 𝟓
Visitors 𝟑𝟐 𝟏𝟐𝟖 𝟓𝟏𝟐 𝟐𝟎𝟒𝟖 𝟖𝟏𝟗𝟐

Solution: Step 1: Determine which model fits the data.


First differences: 𝟗𝟔 , 𝟑𝟖𝟒 , 𝟏𝟓𝟑𝟔 , 𝟔𝟏𝟒𝟒 (Not constant)
Ratios: 𝟒, 𝟒, 𝟒, 𝟒 (Ratios are constant)

Since the ratios of y-values are constant, an exponential function


of the form 𝒚 = 𝒂𝒃𝒙 models the data. The constant ratio is 4,
which is the value of the base.

Step 2: Since we get 𝒚 = 𝒂 • 𝟒𝒙 , 𝒚 = 𝒂 • 𝟒𝒙


next we will find the coefficient 𝒂 𝟑𝟐 = 𝒂 • 𝟒𝟏
using one pair of (𝒙, 𝒚) in the table. 𝒂=𝟖
Finally, we get the function 𝒚 = 𝟖 • 𝟒𝒙 .

Try 5: The table below shows the number of music club members for four
consecutive months. Determine which model fits the data best. Then write
a function that models the data.

Years 𝟏 𝟐 𝟑 𝟒
Members 𝟏𝟓 𝟒𝟓 𝟏𝟑𝟓 𝟒𝟎𝟓

101
Absolute Value Functions

Try 1: (1) Absolute values are always non-negative numbers. True or false? ____
(2) When | 𝒙 | = 𝟒, 𝒙 = _______. (Answer: 𝒙 = +𝟒 or 𝒙 = −𝟒.)
(3) When | 𝒙 | = 𝟎, 𝒙 = _______. (Answer: 𝒙 = 𝟎. )

An absolute value function is a function that contains an algebraic expression


within absolute value symbols such as 𝒇(𝒙) = |𝒙| + 𝟑, 𝒇(𝒙) = | 𝒙 − 𝟏 | .
𝒇(𝒙) = |𝒙| is the parent function of absolute value functions. It is defined as

𝒙 , if 𝒙 > 𝟎
𝒇(𝒙) = |𝒙| = 𝟎 , if 𝒙 = 𝟎
−𝒙 , if 𝒙 < 𝟎

That is, 𝒇(𝒙) = |𝒙| can be written as

𝒚 = 𝒙 , if 𝒙 ≥ 𝟎
𝒚 = −𝒙, if 𝒙 < 𝟎

We can see that the absolute value function can be defined by two or more
expressions. So an absolute value function is a piecewise-defined function.
Since absolute values are always non-negative numbers,
the range of the function 𝒇(𝒙) = |𝒙| is 𝒚 ≥ 𝟎, actually |𝒙| ≥ 𝟎 .
The domain of an absolute value function is the set of all real numbers, which
is −∞ < 𝒙 < +∞ . An absolute value function always has a V-shaped graph.

𝒚 = 𝒂|𝒙 − 𝒉| + 𝒌 is called the vertex form of an absolute value function.


In the vertex form, the vertex of an absolute value function is (𝒉, 𝒌).
The absolute value functions are very similar with the quadratic functions.
Think about 𝒚 = 𝒂(𝒙 − 𝒉)𝟐 + 𝒌, the vertex form of a quadratic function.
102
𝒚 = 𝒂|𝒙 − 𝒉| + 𝒌 can be got by shifting the function 𝒇(𝒙) = |𝒙|.
The 𝒉 represents a horizontal shift. The 𝒌 represents a vertical shift.

Try 2: Find the vertex of each of the following absolute value functions.

A ________ B ________

C ________ D ________

Try 3: Find the minimum value and the range of each of the above functions.

A B C D
Minimum
Range

103
In absolute value function 𝒚 = 𝒂|𝒙| , the leading coefficient 𝒂 affects the opening
direction and opening width. It is similar with 𝒚 = 𝒂𝒙𝟐 . The greater | 𝒂 | is, the
narrower the V-shaped graph is. And the graph of 𝒚 = −|𝒙| opens downward.

Try 4: Compare to the function f(𝒙) = |𝒙| , Fill in the blanks with >, = , < .

A 𝒂 __0 and | 𝒂 |__1 B 𝒂 __ 0 and | 𝒂 | __ 1

C 𝒂 ___ 0 D 𝒂 ___ 0

Try 5: Fill in the blanks.


(1) For 𝒇(𝒙) = −𝟐|𝒙| , the maximum is _______; the range is ____________.

(2) For 𝒇(𝒙) = −|𝒙| , the maximum is _______; the domain is ___________.

104
Example 1: How to graph 𝒚 = 𝒇(𝒙) = 𝟐| 𝒙 + 𝟒 | − 𝟑 on the coordinate plane?

Hint: To graph an absolute value function,


Step 1: Find the vertex first. It’s (𝒉, 𝒌).
Step 2: Set the vertex as the center point,
and find four more points that are
symmetric and close to the vertex.
Usually use five points to graph.
Step 3: Plot the points and graph them
with straight lines.

The following are the points we choose. The vertex is the center.
Can you find the missing values 𝒙 −𝟖 −𝟔 −𝟒 −𝟐 𝟎
according to the symmetry?
𝒚 −𝟑 𝟏 𝟓

Solution: Step 1: For 𝒚 = 𝒇(𝒙) = 𝟐| 𝒙 + 𝟒 | − 𝟑, the vertex is (−𝟒, −𝟑).

Step 2: Find four more points close to the vertex.


When 𝒙 = 𝟎, 𝒚 = 𝟐| ( 𝟎 ) + 𝟒 | − 𝟑 = 𝟓.
When 𝒙 = −𝟐, 𝒚 = 𝟐|(−𝟐) + 𝟒 | − 𝟑 = 𝟏.
When 𝒙 = −𝟔, 𝒚 =______________________.
When 𝒙 = −𝟖, 𝒚 = ______________________.

Step 3: The selected points to graph the function are in the above table.
Plot the five points and connect them with straight lines.
The above is the graph of 𝒇(𝒙) = 𝟐| 𝒙 + 𝟒 | − 𝟑 .

Try 6: (1) For the above function 𝒇(𝒙) = 𝟐| 𝒙 + 𝟒 | − 𝟑, the minimum value
is ________; the range is ____________. the domain is _______________.
(2) For the above function, the graph is symmetric about the line 𝒙 =______.

105
𝟏
Try 7: Please graph 𝒇(𝒙) = | 𝒙 + 𝟑 | on the coordinate plane.
𝟐

𝒙 𝒚

106
Try 8: Please graph 𝒇(𝒙) = −| 𝒙 − 𝟐 | + 𝟒 on the coordinate plane.

𝒙 𝒚

107
Example 2: Graph 𝒇(𝒙) = | 𝟐𝒙 − 𝟔 | on the coordinate plane.

Solution: First, we need find the vertex


by rewriting the function in vertex form.

𝒚 = | 𝟐𝒙 − 𝟔 | (Factor out 2.)

= 𝟐| 𝒙 − 𝟑 | (In vertex form.)

Now, we get that the vertex is (𝟑, 𝟎).

𝒙 𝟏 𝟐 𝟑 𝟒 𝟓

𝒚 𝟒 𝟐 𝟎 𝟐 𝟒

Try 9: For the graph of 𝒚 = | 𝟐𝒙 − 𝟔 | and the graph of 𝒚 = | 𝒙 − 𝟔 |, do they


have the same vertex? ______ Do they have the same opening width? ______

Try 10: Please graph 𝒇(𝒙) = | 𝟑𝒙 + 𝟔 | on the coordinate plane. The axis of
symmetry of the graph is the line of 𝒙 =______.

108
Try 11: Match each function with its graph.
𝟏
(1) 𝒚 = | 𝒙 − 𝟐 | − 𝟒 _____ (2) 𝒚 = | 𝟒𝒙 + 𝟏𝟐 | _____
𝟑
𝟏
(3) 𝒚 = − | 𝒙 + 𝟑 | + 𝟓 _____ (4) 𝒚 = | 𝟒𝒙 | − 𝟕 _____
𝟐

A B

C D

Try 12: (1) If the vertex is (𝟔, −𝟖), then the function 𝒚 = |𝒙 − 𝒉| + 𝒌 is
______________________, where 𝒂 =______.
(2) If the vertex is (−𝟏, 𝟓) and 𝒂 = 𝟑, then the function 𝒚 = 𝒂|𝒙 − 𝒉| + 𝒌
is ____________________.

109
Try 13: (1) If we shift the graph of 𝒚 = 𝟐| 𝒙 − 𝟑 | − 𝟕 up five units, then we
get the graph of the function _____________________ (In vertex form).
(A) 𝒚 = 𝟐| 𝒙 − 𝟑 | − 𝟐 (B) 𝒚 = 𝟐| 𝒙 − 𝟑 | − 𝟏𝟐

(C) 𝒚 = 𝟐| 𝒙 − 𝟏 | − 𝟕 (D) 𝒚 = 𝟐| 𝒙 − 𝟓 | − 𝟕

(2) If we shift the graph of 𝒚 = 𝟓| 𝒙 + 𝟔 | + 𝟖 to the left 5 units, then we get


the graph of the function _____________________ (In vertex form).
(A) 𝒚 = 𝟓| 𝒙 + 𝟏𝟏 | + 𝟖 (B) 𝒚 = 𝟓| 𝒙 + 𝟏 | + 𝟖
(C) 𝒚 = 𝟓| 𝒙 + 𝟔 | + 𝟏𝟑 (D) 𝒚 = 𝟓| 𝒙 + 𝟔 | + 𝟑

Try 14: Please rewrite the following functions in vertex form.


(1) 𝒚 = | 𝟓𝒙 − 𝟐𝟎 | + 𝟏𝟓 ___________________

(2) 𝒚 = 𝟐| 𝟐𝒙 + 𝟖 | − 𝟔 ___________________

Try 15: (1) For the function 𝒚 = | 𝒙 − 𝟓 | + 𝟐 , the vertex is ________.


The minimum value is __________, which occurs at __________.
The graph is symmetric about the line __________. The y-intercept is ______.

(2) For the function 𝒚 = | 𝟔𝒙| + 𝟕 , the vertex is ________.


The minimum value is __________, which occurs at __________.
The axis of symmetry of the graph is __________. The y-intercept is ______.

Try 16: Given the graph of an


absolute value function on the right,
please write the function of the graph
in vertex form.

110
Practice 7

1. The parabola of a quadratic function 𝒚 = 𝒂𝒙𝟐 + 𝒃𝒙 + 𝒄 goes through the


points of (𝟎, 𝟏), (𝟐, 𝟎) and (−𝟑, 𝟎). What is the quadratic function of
the parabola?

2. The graph of a quadratic function has the vertex of (𝟏, 𝟑), and it goes
through the points of (𝟐, 𝟕). Please find the equation of the quadratic
function from the graph.

3. On the right is the graph of a


quadratic function. Please find
the equation of the quadratic
function from the graph.

111
4. Determine which kind of functions model the following data best.
Then write an equation for the function that models the data.

𝒙 −𝟏 𝟎 𝟏 𝟐 𝟑

𝒚 −𝟒 −𝟔 −𝟔 −𝟒 𝟎

5. Determine which kind of functions best model the data in each table.
Then write an equation for the function that models the data.

(1) 𝒙 𝒚 (2) 𝒙 𝒚
𝟎 𝟑 −𝟑 𝟏
𝟏 𝟔 −𝟐 𝟏. 𝟓
𝟐 𝟏𝟐 −𝟏 𝟐
𝟑 𝟐𝟒 𝟎 𝟐. 𝟓
𝟒 𝟒𝟖 𝟏 𝟑

112
6. Find the vertex and minimum value of each of the following functions.

The vertex is __________. The vertex is __________.


The minimum value is ______. The minimum value is ______.

7. (1) For the function 𝒚 = 𝟑| 𝒙 + 𝟏𝟐 | , the vertex is __________.


The minimum value is __________, which occurs at __________.
The graph is symmetric about the line __________. The y-intercept is _____.

(2) For the function 𝒚 = | 𝟑𝒙 + 𝟏𝟐 | , the vertex is _________.


The minimum value is __________, which occurs at __________.
The axis of symmetry of the graph is __________. The y-intercept is ______.

8. On the right is the graph of an


absolute value function. Please
find the equation of the
function from the graph.

113
9. Which kind of functions model each set of ordered pairs best, a linear function,
a quadratic function or an exponential function?
(1) (𝟎, 𝟑), (𝟏, 𝟑. 𝟓), (𝟐, 𝟒), (𝟑, 𝟒. 𝟓), (𝟒, 𝟓). _____________

(2) (−𝟐, 𝟏𝟏), (−𝟏, 𝟕), (𝟎, 𝟑), (𝟏, −𝟏), (𝟐, −𝟓). _____________

(3) (𝟎, 𝟎), (𝟏, 𝟏. 𝟓), (𝟐, 𝟔), (𝟑, 𝟏𝟑. 𝟓), (𝟒, 𝟐𝟒). _____________

10. The table below shows the value of a used car over time. Which type of
function best models the data? Write an equation to model the data.

Years 𝟎 𝟏 𝟐 𝟑 𝟒
Value($) 𝟏𝟖, 𝟖𝟒𝟎 𝟏𝟔, 𝟎𝟏𝟒 𝟏𝟑, 𝟔𝟏𝟐 𝟏𝟏𝟓, 𝟕𝟎 𝟗, 𝟖𝟑𝟓

114
Chapter Summary

For the graphs of the functions 𝒚 = 𝒂𝒙𝟐 ,


the vertex is the origin (0, 0);
the axis of symmetry is the y-axis.
If 𝒂 > 𝟎 , the parabola opens upward.
If 𝒂 < 𝟎 , the parabola opens downward.
The greater | 𝒂 | is, the narrower the parabola is.
The smaller | 𝒂 | is, the wider the parabola is.

The domain of a function is the set of all values of 𝒙.


The range of a function is the set of all values of 𝒚.

If we shift the graph of 𝒚 = 𝟐𝒙𝟐


to the left three units, we can get
the graph of 𝒚 = 𝟐(𝒙 + 𝟑)𝟐 .
If we further shift the graph of
𝒚 = 𝟐(𝒙 + 𝟑)𝟐 down 4 units,
then we can get the function of
𝒚 = 𝟐(𝒙 + 𝟑)𝟐 − 𝟒
See the graphs on the right.

The function 𝒚 = 𝒂(𝒙 − 𝒉)𝟐 + 𝒌 is called the vertex form


of a quadratic function. In the vertex form, the vertex is (𝒉, 𝒌).
The axis of symmetry of the function is the line of 𝒙 = 𝒉.
And 𝒌 is the minimum or maximum value of the function.
When a > 𝟎 , there is minimum and the range of the quadratic function is 𝒚 ≥ 𝒌.
When a < 𝟎 , there is maximum and the range of the quadratic function is 𝒚 ≤ 𝒌.

115
𝒃 𝟒𝒂𝒄−𝒃𝟐
The vertex of 𝒚 = 𝒂𝒙𝟐 + 𝒃𝒙 + 𝒄 is (− , ).
𝟐𝒂 𝟒𝒂
𝒃
The axis of symmetry is x = − , which goes through the vertex.
𝟐𝒂
𝟒𝒂𝒄−𝒃𝟐 𝒃
The minimum or maximum value is or the y-value when 𝒙 = − .
𝟒𝒂 𝟐𝒂

Equation 𝒙𝟐 − 𝟔𝒙 + 𝟓 = 𝟎 𝒙𝟐 − 𝟔𝒙 + 𝟗 = 𝟎 𝒙𝟐 − 𝟔𝒙 + 𝟏𝟏 = 𝟎

Discriminant 𝒃𝟐 − 𝟒𝒂𝒄 > 𝟎 𝒃𝟐 − 𝟒𝒂𝒄 = 𝟎 𝒃𝟐 − 𝟒𝒂𝒄 < 𝟎

Graph of
Related
Function

Intercepts 2 x-intercepts 1 x-intercept 0 x-intercepts

Real Roots Two Roots One Root None

116
The axis of symmetry of a quadratic function is halfway between any pair of
two symmetric points of the function. If p and q represents the x-coordinates
𝟏
of the two symmetric points, then the axis of symmetry is 𝒙 = (𝒑 + 𝒒) .
𝟐

In the quadratic function 𝒚 = 𝒂𝒙𝟐 + 𝒃𝒙 + 𝒄 (𝒂 ≠ 𝟎) ,


the value of 𝒃 affects the position of the axis of symmetry.

𝟏 𝟏 𝟏
𝒚 = 𝒙𝟐 − 𝟒𝒙 𝒚 = 𝒙𝟐 + 𝟐𝒙 𝒚 = 𝒙𝟐 + 𝟒𝒙
𝟐 𝟐 𝟐

𝒃
Consider the above graphs. Since the axis of symmetry is 𝒙 = − , we get
𝟐𝒂
−𝟒 𝟐 𝟒
𝒙=− 𝟏 =𝟒 𝒙=− 𝟏 = −𝟐 𝒙=− 𝟏 = −𝟒
𝟐• 𝟐• 𝟐•
𝟐 𝟐 𝟐

Notice that the axis of symmetry changes with each change in the 𝒃-value when
the leading coefficient 𝒂 is fixed. When the constant term 𝒄 is missing (𝒄 = 𝟎)
in the quadratic function, the graph goes through the origin (0, 0).

Function Notation

𝒇(𝒙) = 𝒙𝟐 − 𝟔𝒙 + 𝟓

𝒇 is the name This tells you This tells you what


of the function the value of 𝒙 The function does

117
The zero of a function is the x-value that makes the function equal to 0.
To find the zero of a function means to solve the equation 𝒇(𝒙) = 𝟎 for x.

A ball is shot into the air from a platform,


which is 80 feet above the ground.
The ball is launched at 64 feet per second.
The function 𝒉 = −𝟏𝟔𝒕𝟐 + 𝟔𝟒𝒕 + 𝟖𝟎
represents the height of the ball after
𝒕 seconds.

𝒉 = −𝟏𝟔𝒕𝟐 + 𝟔𝟒𝒕 + 𝟖𝟎

Gravity pulls it down, The initial height is 80


The ball is launched at
by about 16 feet per feet above the ground.
an initial velocity of
second squared
64 feet per second

Given two roots of 𝒙𝟏 and 𝒙𝟐 of a quadratic equation 𝒂𝒙𝟐 + 𝒃𝒙 + 𝒄 = 𝟎 ,


we can factor it as:
𝒂𝒙𝟐 + 𝒃𝒙 + 𝒄 = 𝒂(𝒙 − 𝒙𝟏 ) (𝒙 − 𝒙𝟐 )

Model Selection: When the x-values in a set of data pairs have a common
difference, you can analyze data numerically to find the best model.
(1) You can use a linear function to model data pairs with y-values that have
a common difference.
(2) You can use an exponential function to model data pairs with y-values that
have a common ratio.
(3) You can use an quadratic function to model data pairs with y-values that
have a common second difference.

118
Chapter Practice

𝟏
1. The vertex of the function 𝒇(𝒙) = − (𝒙 + 𝟏)𝟐 + 𝟏 is _______.
𝟑
(A) (𝟏 , 𝟏 ) (B) (−𝟏 , 𝟏 )
(C) (𝟏 , −𝟏 ) (D) (−𝟏 , −𝟏 )

2. The zero of the function 𝒇(𝒙) = 𝒙𝟐 − 𝒙 − 𝟔 are _____________.


(A) 𝟐 and 𝟑 (B) −𝟐 and 𝟑
(C) 𝟐 and −𝟑 (D) −𝟐 and −𝟑
𝟏
3. If the function 𝒉(𝒙) = 𝒙𝟐 − 𝟑 is shifted to the left 2 units, then we get
𝟒
the function 𝒋(𝒙), that can be defined by which expression? _________
𝟏 𝟏
(A) (𝒙 − 𝟐)𝟐 − 𝟑 (B) 𝒙𝟐 − 𝟏
𝟒 𝟒
𝟏 𝟏
(C) (𝒙 + 𝟐)𝟐 − 𝟑 (D) 𝒙𝟐 − 𝟓
𝟒 𝟒

4. In the function 𝒇(𝒙) = (𝒙 − 𝟓)𝟐 + 𝟐, the minimum value is _______.


The minimum value occurs when x is _______.
(A) 𝟓 (B) −𝟓 (C) 𝟐 (D) −𝟐

5. If the graph of a function opens downward and its vertex is (−𝟑, 𝟒),
then the range of the function is _________.
(A) greater than or equal to −𝟑 (B) greater than −𝟑
(C) smaller than or equal to 𝟒 (D) smaller than 𝟒

6. The expression −𝟓𝒕𝟐 + 𝟏𝟒𝒕 + 𝟑 represents the height, in meters, of a ball


𝒕 seconds after launch. The initial height of the ball, in meters, is ______.
(A) 𝟎 (B) 𝟑 (C) 𝟓 (D) 𝟏𝟒
𝟏
7. Given the vertex of the function 𝒇(𝒙) = − (𝒙 + 𝒂)𝟐 + 𝒃 is (−𝟏, −𝟑),
𝟐
𝟏
then function is 𝒇(𝒙) = − (𝒙 + ____)𝟐 + _____.
𝟐

119
8. Please find the vertex of the function 𝒚 = −𝒙𝟐 − 𝟒𝒙 first. And then graph
𝒚 = −𝒙𝟐 − 𝟒𝒙 on the coordinate plane.

𝒙 𝒚

120
9. Let 𝒉 be the function represented
by the graph on the right.
Let 𝒈 be the function such that
𝟏
𝒈(𝒙) = − 𝒙𝟐 − 𝟐𝒙 + 𝟒 .
𝟑

Determine which function has the


larger maximum value. ________
Justify your answer.

10. Below is a table of values of a quadratic function. Please find:


𝒙 −
𝟏
𝟎 𝟏 𝟐 𝟐
𝟏
𝟐 Can you fill in the
𝟐
𝟏 missing value?
𝒚 −𝟐
𝟐
𝟎 𝟐 𝟎

(1) The vertex of the quadratic function in the above table is _________.
(2) The y-intercept of the graph is __________.
(3) The zeros of the function are __________.
(4) The axis of symmetry of the graph is __________.

11. The parabola of the quadratic function 𝒚 = 𝒙𝟐 − 𝟐𝒙 + 𝟑 must intersect


with ___________. (A) the x-axis (B) the y-axis
(C) both the x-axis and the y-axis

12. Below is the table of values of a quadratic function. Does the graph of the
function has the intercepts with x-axis? ________
𝒙 −𝟏 𝟎 𝟏 𝟐 𝟑 𝟒
𝒈(𝒙) −𝟖 −𝟓 −𝟒 −𝟓 −𝟖 −𝟏𝟑

121
13. Find the axis of symmetry of the quadratic function 𝒇(𝒙) = 𝟐𝒙𝟐 − 𝟖𝒙 − 𝟏 .

14. If the parabola of a quadratic function goes through two points: (𝟏 , −𝟒 )


and (−𝟓 , −𝟒 ), find the axis of symmetry of the parabola.

15. If the axis of symmetry of the function 𝒚 = 𝒙𝟐 + (𝒎 − 𝟐)𝒙 + 𝟕 is the


𝟏
line of 𝒙 = , then what is the value of m ?
𝟐

16. If the vertex of the function 𝒚 = 𝟐𝒙𝟐 + 𝒃𝒙 + 𝒄 is (𝟏, −𝟐), then


(1) what is the value of b ? (2) what is the value of c ?

17. On the right is the graph


of the quadratic function
𝒚 = 𝒂𝒙𝟐 + 𝒃𝒙 + 𝒄 (𝒂 ≠ 𝟎).

Please determine 𝒂 ____ 𝟎,


𝒃 ____ 𝟎,
𝒄 ____ 𝟎.

(A) > (B) = (C) <

122
18. The parabola of the quadratic function 𝒚 = 𝒂𝒙𝟐 + 𝒃𝒙 + 𝒄 (𝒂 ≠ 𝟎) has
two x-intercepts: (−𝟐 , 𝟎 ) and (−𝟔 , 𝟎). What is the axis of symmetry
of the parabola.

19. If the zeros of a quadratic function are −𝟓 and 𝟕 , what is the axis of
symmetry of the function? Justify your answer.

20. The parabola of the quadratic function 𝒚 = 𝒂𝒙𝟐 + 𝒃𝒙 + 𝒄 (𝒂 ≠ 𝟎) has


two x-intercepts: (−𝟑 , 𝟎 ) and (𝟒 , 𝟎). The related equation could be
_______________ .
(A) 𝒚 = 𝒂(𝒙 − 𝟑)(𝒙 − 𝟒) (B) 𝒚 = (𝒂𝒙 − 𝟑)(𝒙 + 𝟒)

(C) 𝒚 = 𝒂(𝒙 + 𝟑)(𝒙 − 𝟒) (D) 𝒚 = (𝒂𝒙 + 𝟑)(𝒙 − 𝟒)

21. The graph of 𝒚 = 𝒂𝒙𝟐 passes through the point of (𝟐, 𝟏𝟎).
(1) Find the value of 𝒂. (2) Find the function.

22. Find the axis of the symmetry, the vertex and the opening direction of
each function. (1) 𝒚 = 𝒙(𝟒 − 𝒙) (2) 𝒚 = (𝟑 + 𝒙)(𝟏 − 𝒙)

123
23. Which of the following quadratic function has the largest maximum? _______
(A) 𝒚 = (𝟏 + 𝒙)(𝟏 − 𝒙) (B) 𝒚 = −𝒙𝟐 − 𝟒𝒙 + 𝟐

(C) 𝒚 = −(𝒙 + 𝟏)𝟐 + 𝟑 (D) 𝒚 = −| 𝒙 − 𝟒| + 𝟓

𝒙 𝒚
𝟎 𝟓

−𝟏 𝟖

−𝟐 𝟗

−𝟑 𝟖

−𝟒 𝟓

(E) (F)

24. Which of the following quadratic function has the least minimum? _______
(A) 𝒚 = 𝒙𝟐 − 𝟒𝒙 − 𝟏 (B) 𝒚 = (𝒙 + 𝟏)(𝒙 + 𝟓)

(C) 𝒚 = 𝟐(𝒙 + 𝟒)𝟐 − 𝟑 (D) 𝒚 = | 𝒙 − 𝟐 | − 𝟓

𝒙 𝒚
−𝟏 𝟎

𝟎 −𝟒

𝟏 −𝟔

𝟐 −𝟔

𝟑 −𝟒

(E) (F)

124
25. Please write the function 𝒇(𝒙) = 𝒙𝟐 + 𝟔𝒙 + 𝟒 in vertex form by
completing the square.

26. The function 𝒇(𝒙) = 𝒙𝟐 − 𝟖𝒙 − 𝟓 can be written in vertex form as ______.


(A) 𝒇(𝒙) = (𝒙 + 𝟒)𝟐 − 𝟏𝟏 (B) 𝒇(𝒙) = (𝒙 + 𝟖)𝟐 − 𝟏𝟑
(C) 𝒇(𝒙) = (𝒙 − 𝟒)𝟐 − 𝟐𝟏 (D) 𝒇(𝒙) = (𝒙 − 𝟖)𝟐 − 𝟑

27. The function 𝒇(𝒙) = −𝟑𝒙𝟐 − 𝟏𝟖𝒙 − 𝟐𝟗 can be written in vertex form
as ________________. Please write down the process.

(A) 𝒇(𝒙) = −𝟑(𝒙 + 𝟑)𝟐 − 𝟐 (B) 𝒇(𝒙) = −(𝟑𝒙 + 𝟗)𝟐 − 𝟓𝟐


(C) 𝒇(𝒙) = −𝟑(𝒙 − 𝟑)𝟐 − 𝟐 (D) 𝒇(𝒙) = −(𝟑𝒙 + 𝟗)𝟐 − 𝟏𝟏𝟎

125
28. If the equation 𝒙𝟐 + 𝒃𝒙 + 𝒄 = 𝟎 has two roots 𝒙𝟏 = −𝟕, 𝒙𝟐 = −𝟓, then
we can factor 𝒙𝟐 + 𝒃𝒙 + 𝒄 as ________.
(A) (𝒙 − 𝟕)(𝒙 − 𝟓) (B) (𝒙 + 𝟕)(𝒙 + 𝟓)
(C) (𝒙 − 𝟕)(𝒙 + 𝟓) (D) (𝒙 + 𝟕)(𝒙 − 𝟓)

29. The x-intercepts of the quadratic function 𝒚 = (𝒙 − 𝟐)(𝒙 + 𝟑) are ______.


The x-intercepts of the quadratic function 𝒚 = 𝟓(𝒙 − 𝟐)(𝒙 + 𝟑) are _____.
(A) (−𝟐 , 𝟎 ) and (𝟑 , 𝟎) (B) (𝟐 , 𝟎 ) and (−𝟑 , 𝟎)
(C) (𝟎, −𝟐 ) and (𝟎 , 𝟑) (D) (𝟎 , 𝟐) and (𝟎 , −𝟑)

30. If the equation 𝟓𝒙𝟐 + 𝒃𝒙 + 𝒄 = 𝟎 has two roots 𝒙𝟏 = −𝟐, 𝒙𝟐 = 𝟑, then


we can factor 𝟓𝒙𝟐 + 𝒃𝒙 + 𝒄 as ________.
(A) 𝟓(𝒙 − 𝟐)(𝒙 + 𝟑) (B) (𝟓𝒙 − 𝟐)(𝒙 + 𝟑)
(C) 𝟓(𝒙 + 𝟐)(𝒙 − 𝟑) (D) (𝟓𝒙 + 𝟐)(𝒙 − 𝟑)

31. Below is the table of values of 𝒇(𝒙) = 𝒂𝒙𝟐 + 𝒃𝒙 + 𝒄 . The data pairs are
close to the x-intercept. The root of 𝒂𝒙𝟐 + 𝒃𝒙 + 𝒄 = 𝟎 is 𝒙 = _______.
𝒙 𝟔 𝟔. 𝟏 𝟔. 𝟐 𝟔. 𝟑

𝒇(𝒙) −𝟏 −𝟎. 𝟐𝟗 𝟎. 𝟒𝟒 𝟏. 𝟏𝟗

32. Among the following equations, which one has one real root? _______
which one has two real roots ? _______
(A) (𝒙 + 𝟒)𝟐 − 𝟓 = 𝟎 (B) (𝒙 + 𝟒)𝟐 = 𝟎
(C) (𝒙 + 𝟒)𝟐 + 𝟓 = 𝟎 (D) 𝒙𝟐 + 𝟒 = 𝟎
33. Without solving each equation, determine the nature of the roots.
(A) 𝒙𝟐 − 𝟑𝒙 + 𝟓 = 𝟎 _______ (B) 𝟑𝒚𝟐 − 𝟏𝟎𝒚 + 𝟏 = 𝟎 _______

(C) 𝒙𝟐 − 𝟓𝒙 + 𝟒 = 𝟎 _______ (D) 𝒚𝟐 − 𝟏𝟎𝒚 + 𝟐𝟓 = 𝟎 _______

126
34. Below are the tables of the quadratic functions. How many real roots do the
related equations have? (1) __________ (2) __________

(1) 𝒙 −𝟏 𝟎 𝟏 𝟐 𝟑 𝟒
𝒈(𝒙) 𝟔 𝟑 𝟐 𝟑 𝟔 𝟏𝟏

(2) 𝒙 −𝟓 −𝟒 −𝟑 −𝟐 −𝟏 𝟎
𝒇(𝒙) 𝟑 −𝟏 −𝟑 −𝟑 −𝟏 𝟑

𝟏
35. If each function of 𝒇(𝒙) = 𝟑𝒙𝟐 and 𝒈(𝒙) = |𝟑𝒙| is multiplied by ,
𝟓
then which statement about these new functions is true? _______
(1) The graphs of both 𝒇(𝒙) and 𝒈(𝒙) become wider.
(2) The graphs of both 𝒇(𝒙) and 𝒈(𝒙) become narrower.
(3) The graphs of both 𝒇(𝒙) and 𝒈(𝒙) shift vertically.
(4) The graphs of both 𝒇(𝒙) and 𝒈(𝒙) shift horizontally.

36. The zero of the function 𝒇(𝒙) = 𝟐𝒙𝟐 − 𝟔𝒙 − 𝟖 are _____________.


(A) 𝟏 and 𝟒 (B) −𝟏 and 𝟒
(C) 𝟏 and −𝟒 (D) −𝟏 and −𝟒

37. Given the function 𝒇(𝒙) = 𝒙𝟐 − 𝟓𝒙 + 𝟑, what is the value of 𝒇(𝟑) ? ______
(A) 𝟑 (B) −𝟑
(C) 𝟕 (D) −𝟕

38. If the domain of the function 𝒇(𝒙) = 𝟐𝒙𝟐 − 𝟔 is 𝟏 ≤ 𝒙 ≤ 𝟓, then the


range is _______________.
(A) 𝟏 ≤ 𝒙 ≤ 𝟓 (B) −𝟒 ≤ 𝒚 ≤ 𝟒
(C) 𝟐 ≤ 𝒙 ≤ 𝟔 (D) −𝟒 ≤ 𝒚 ≤ 𝟒𝟒

39. Which point is not on the graph represented by 𝒚 = 𝒙𝟐 + 𝟐𝒙 − 𝟖 ?


(A) (𝟏, −𝟓) (B) (𝟎 , 𝟐)
(C) (−𝟏, −𝟗) (D) (𝟎, −𝟖)

127
𝟏 𝟏
40. Graphing the quadratic function 𝒚 = 𝒙𝟐 − 𝒙 − 𝟑. From the graph, we can
𝟐 𝟐
find that: (1) When 𝒙 =___________, the value of y is 0.
(2) When 𝒚 =___________, the value of x is 0.
(3) When 𝒙 is ______________, the value of y < 0.
(4) When 𝒙 is ______________, the value of y > 0.

(A) −𝟐 < 𝒙 < 𝟑 (C) −𝟐 > 𝒙 and 𝒙 > 𝟑 (E) 𝟐 (G) −𝟐


(B) −𝟐 ≤ 𝒙 ≤ 𝟑 (D) −𝟐 < 𝒙 and 𝒙 > 𝟑 (F) 𝟑 (H) −𝟑

128
41. A Textile manufacturer has daily production costs of the function
𝑪 = 𝟏𝟎, 𝟎𝟎𝟎 − 𝟏𝟏𝟎𝒙 + 𝟎. 𝟎𝟒𝟓𝒙𝟐 , where 𝑪 is the total cost (in dollars) and 𝒙
is the number of units produced. How many units should be produced each day
to yield a minimum cost?

42. The value of Jennifer’s stock portfolio is given by the function


𝑽(𝒕) = 𝟓𝟎 + 𝟕𝟑𝒕 − 𝟑𝒕𝟐 , where V is the value of the portfolio in hundreds of
dollars and t is the time in months. How much money did Jennifer start with?
When will the value of Jennifer’s portfolio be at a maximum?

43. Given the functions 𝒈(𝒙) = 𝒙𝟐 + 𝟐𝒙 − 𝟏 and 𝒇(𝒙) = 𝒙 + 𝟓, find the values
of 𝒙 for which 𝒈(𝒙) = 𝒇(𝒙). Please find the answer algebraically.

129
44. Below is the path of water from a water fountain. The maximum height of the
path is 6 feet, which occurs at a horizontal distance of 2 feet. The water fountain
itself is 4 feet high. Find the equation of the function from the graph. ________
𝟏
(A) 𝒇(𝒙) = − (𝒙 − 𝟐)𝟐 − 𝟔
𝟒
𝟏
(B) 𝒇(𝒙) = − (𝒙 + 𝟐)𝟐 + 𝟔
𝟒
𝟏
(C) 𝒇(𝒙) = − (𝒙 − 𝟐)𝟐 + 𝟔
𝟐
𝟏
(D) 𝒇(𝒙) = − (𝒙 + 𝟐)𝟐 − 𝟔
𝟐

45. Determine which kind of functions model the following data best.
Then write an equation for the function that models the data.

𝒙 −𝟏 𝟎 𝟏 𝟐 𝟑

𝒚 −𝟔 𝟎 𝟐 𝟎 −𝟔

130
46. Given the parabola of a quadratic function goes through the points of
(𝟎, −𝟐), (𝟏, 𝟎) and (𝟐, 𝟑), write an equation to model the data.

47. A conservation organization collected the data, f(t) on the number of crabs ,
in thousands, in a river over years, t, shown in the graph below. Which function
best represents the graph of f(t) ? _____________

(A) 𝒇(𝒕) = 𝟏. 𝟑𝒕𝟐 − 𝟎. 𝟑𝒕 − 𝟐

(B) 𝒇(𝒕) = 𝟐(𝟐)𝒕


(C) 𝒇(𝒙) = 𝟐𝒕 + 𝟐

131
48. Peter hits a golf ball from the ground. The height of the ball above the ground
𝟏 𝟖
is modeled by the function 𝒉(𝒙) = − 𝒙𝟐 + 𝒙 , where 𝒙 represents the
𝟒𝟖 𝟗
horizontal distance, since the ball was hit. Both 𝒉(𝒙) and x are measured in meters.
(1) What is the maximum height the golf ball can reach?
(2) How far is the golf ball hit out, when it hits the ground?
(3) Please graph the function on the following coordinate plane.

132
49. A toy rocket was launched from the ground. The height of the rocket,
𝟏
in meters, is modeled by the function 𝒉(𝒕) = − 𝒕𝟐 + 𝟐𝒕 , where 𝒕
𝟓
represents the time in seconds, since the rocket was launched.
(1) How long it takes the object to hit the ground?
(2) What is the average rate of change of the function between
5 and 10 seconds?

133
50. A player throws a basketball toward the hoop. The initial height of the
𝟐𝟎
basketball is meters. The player is 8 meters away from the center of
𝟗
the hoop. The hoop is 3 meters high. At the horizontal distance of 4 meters,
the basketball reaches its maximum height, which is 4 meters high.
(1) Please find the function that could model the path of the basketball.
(2) Will the ball pass through the hoop?

134
51. Which point is not on the graph represented by 𝒚 = 𝒙𝟐 + 𝟑𝒙 − 𝟒 ?
(A) (𝟎, −𝟒) (B) (𝟏, 𝟎)
(C) (𝟑, −𝟒) (D) (𝟐, 𝟔)

52. Given the function 𝒇(𝒙) = 𝒙𝟐 + 𝟖𝒙 + 𝟏𝟐 , find the coordinates of


the vertex of the parabola defined by 𝒈(𝒙) = 𝒇(𝒙 − 𝟑).

53. On the right is the graph of an


absolute value function. Please
write a function that can model
the graph.

135

You might also like